SlideShare une entreprise Scribd logo
1  sur  96
The EKG in: Metabolic and Electrolyte Disorders MDFMR/UNECOM M. LaCombe May 6, 2009
The normal EKG waveform
A normal EKG
The frontal plane (QRS) axis:
Elderly female admitted with obtundation. What electrolyte abnormality is strongly suggested?     a)   Hyponatremia b)   Hypernatremia c)   Hyperkalemia d)   Hypokalemia e)   Hypercalcemia
Hyperkalemia (K+ = 8.7 mEq/L) secondary to acute renal failure. The ECG shows symmetrically peaked ("tented") T waves associated with potassium levels in excess of 6 mEq/L. The tracing also shows broad and flattened sinus P waves that may precede frank sino-ventricular conduction seen with severe hyperkalemia (i.e., conduction from the sinus node to the ventricles through specialized inter-nodal tissue without atrial depolarization). This conduction pattern may simulate a junctional rhythm. The narrow QRS complex in this tracing is somewhat atypical for severe hyperkalemia. Milder degrees of hyperkalemia may actually facilitate AV conduction. Note that T wave peaking with hyperkalemia is a relative finding: the absolute magnitude of the T waves cannot be used to rule in or rule out hyperkalemia. Indeed, in some cases of hyperkalemia with left ventricular hypertrophy, the T wave peaking may affect inverted T waves or cause relative normalization. The presence of relatively low voltage and sinus bradycardia in this clinical setting should also raise consideration of hypothyroidism/myxedema, which by itself usually also causes T wave flattening.
This is the admitting ECG of a previously healthy 49-year-old man who presented with progressive muscle weakness and constipation. He had no chest pain or dyspnea. The ECG is most consistent with which ONE of the following diagnoses?  a)   Hypokalemia b)   Hyperkalemia c)   Hypocalcemia d)   Hypercalcemia e)   Hypothyroidism
The ECG shows sinus rhythm with the following abnormalities a very short ST segment with a consequently short QT interval (about 300 msec here). The differential diagnosis of a short QT (lower limits are not well-defined) is much narrower than that of prolonged QT. The two chief causes of a short QT are hypercalcemia and digoxin therapy (associated with characteristic "scooping" of the ST-T complex). A third and relatively rare cause is hereditary short QT ("channelopathy"-related) that may be associated with ventricular arrhythmia and sudden cardiac arrest. Cardiac arrhythmias, however, are unusual with hypercalcemia; but AV block, sinus arrest, sino-atrial block, ventricular tachycardia, and cardiac arrest have been reported, usually in patients receiving rapid IV injections of calcium. This patient's serum calcium was 16 mg/dl. He was found to have hyperparathyroidism; at surgery, he had successful resection of a parathyroid carcinoma, an ususual cause of hypercalcemia.
The patient is an elderly man who presented to the emergency ward with dizziness and new renal failure.
Dx: Hyperkalemia (7.6 mEq/L) secondary to renal failure. The ECG demonstrates findings consistent with severe hyperkalemia -- most importantly marked widening of the QRS complex. The QRS complex here shows a left bundle branch block (LBBB) morphology with left axis deviation. However, the QRS duration (about 240 ms) is much wider than seen with an "uncomplicated" LBBB or just with left anterior fascicular block due to intrinsic conduction disease. There is also peaking of the T waves with prolongation of the PR interval and flattening of the P waves. If the hyperkalemia is left untreated, the ECG will progress to a sinusoidal pattern and eventually asystole with subsequent hemodynamic collapse and death.
A 38-yr-old woman with weakness. Previous ECG was normal and she was on no medications. What is the most likely diagnosis?     a)   Hypercalcemia b)   Hypernatremia c)   Hypokalemia d)   Hypocalcemia e)   Hyponatremia
The ECG shows sinus bradycardia with diffuse T wave flattening or inversions, and markedly prominent U waves. These are best seen in leads V2 and V3, but are essentially invisible in lead aVL. The two most common causes of this finding are 1) Hypokalemia (K+ here was 2.4 mEq/L) and 2) Drugs, especially the class 1A antiarrhythmics (like quinidine, procainamide, disopyramide) and related agents (like the phenothiazines and tricyclics), etc. Patients with hereditary (congenital) long QT syndromes due to "channelopathies" may show a similar finding. This ventricular repolarization prolongation pattern is of great importance because it identifies patients at high risk of torsade de pointes type of polymorphic ventricular tachycardia.
This ECG "speaks" major clues about the underlying major medical condition in this 77-year-old woman. What does it say?
ECG shows sinus rhythm with left ventricular hypertrophy (LVH), left atrial abnormality and tall peaked T waves best seen in the precordial leads (with non-specific infero-lateral ST depressions). Also present are left axis deviation consistent with left anterior fascicular block and a borderline long QT. Putting it all together, the peaked T waves indicate increased K+ (6.3 mEq/L) with LVH suggesting renal failure associated with hypertension, which was the case here. The long QT with stretchout ST segment phase goes with decreased calcium seen with renal failure as well. ST depressions could be from LVH or primary ischemia, in addition to multiple other causes. So when you see the triad of "tented" T waves (hyperkalemia) with long QT (ST segment prolongation consistent with hypocalcemia) and LVH (hypertension) the patient virtually always has renal failure.
A 65-year-old man. What should you do before calling cardiology fellow for a "hyperacute myocardial infarction" here?
This is classic hyperkalemia with tall peaked T waves, along with PR prolongation and underlying left ventricular hypertrophy (LVH) (patient has renal disease with hypertension). Note QRS prolongation also seen with moderate-severe hyperkalemia. Potassium here was 9.6mEq/L. Not all tall positive T waves are "hyperacute." The latter term should be reserved for increased T wave positivity secondary to transmural ischemia. Major clues include narrowness (tenting) of T waves along with other findings noted above.
30-yr-old man, not on medication. What waveform is prominent? What is the likely diagnosis, assuming prior normal ECG? 
There are very (!) prominent U waves with Q-T(U) prolongation. He had severe hypokalemia (1.5 mEq/L) due to diarrhea. Calcium was normal, as was magnesium (remember pure hypocalcemia prolongs the ST segment primarily but does not give large U waves.) Obviously, this acquired-type long-QT(U) syndrome puts the subject at risk for torsade de pointes which he fortunately did not have before coming to hospital. The same type ECG may occur with drug-related QT(U) prolongation syndromes and with hereditary long QT(U) syndromes related to "channelopathies," including the Romano-Ward and Jervell Lange-Nielsen syndromes.
Paris
34-year-old male. What's going on here?
ECG shows classic findings of acute/hyperacute anterior wall Q wave myocardial infarction (MI), with reciprocal inferior ST depressions. The distribution of changes is consistent with a proximal left anterior descending (LAD) occlusion which was confirmed at cardiac catheterization and treated with angioplasty/stent. Premature atherosclerosis here was associated with multiple risk factors for coronary artery disease:hypertension, hyperlipidemia, family history and tobacco.
Minimalist/optimal medicine quiz. If you could do only one lab test, what would it be in this case?
Check the calcium (and yes, of course albumin). It was low here at 8.2 mg/dl with minimally low albumin in patient post thyroidectomy. ECG shows QT prolongation. Most important is that QT is long in this case because the ST segment is stretched out. This relates to prolongation of the plateau phase of action potential which is prolonged with hypocalcemia. Ventricular repolarization can also be long because of 1) wide QRS; 2) flat, wide T waves (e.g. quinidine, sotalol, hypokalemia, etc); and/or 3) prominent U waves melding with T wave. The latter two syndromes, representing dispersion of ventricular refractoriness, are most likely to be associated with torsade de pointes.
What's up here?
Serum K+. It was 7.7 mEq/L. Note the relatively tall peaked (tented) T waves that are a classic sign of mild-moderate hyperkalemia. With more severe elevations, the QRS will widen, as will the PR, and the P wave amplitude will diminish, resulting in a junctional or idioventricular-type rhythm eventually.
35-yr-old: If you could only obtain one lab test what would it be?
Classic (!) ECG for hypercalcemia (13.9 mg/dl here). Note the very abbreviated ST segment such that the T wave looks like it takes off directly from QRS in some leads. High take-off of ST segment in V2/V3 is also sometimes seen in this context. Hypocalcemia is just the opposite: prolonged QT due to long ST segment (corresponding to changes in action potential plateau phase duration).
51-yr-old male; what is the key ECG finding and its differential diagnosis?
The most notable finding here is a markedly prolonged QT(U) interval, best seen in the lateral precordial leads. The differential diagnosis of this important repolarization abnormality centers on 1) electrolyte disorders, especially hypokalemia and hypomagnesemia (the K+ here was at most 3.6 mEq/L, and probably lower, with a borderline low Mg++ of 1.6 mEq/L; the serum calcium was normal); and 2) drug effects (e.g., quinidine, sotalol, amiodarone, tricyclic antidepressants,etc). Hereditary long QT syndromes are much rarer. Hypocalcemia by itself stretches out the ST segment but does not produce prominent U waves. The long QT(U) with hypokalemia/hypomagnesemia is associated with an increased risk of torsade de pointes in this setting.
This ECG is most consistent with:    a)   Acute anterior myocardial infarction (MI) b)   Pericarditis/tamponade c)   Renal failure d)   Tricyclic antidepressant overdose e)   Myxedema coma
c) There is classic evidence of hyperkalemia with tall, narrow T waves, most evident in leads V3-V4, along with some QRS prolongation. These "tented T waves" are different from the broader hyperacute T waves of acute MI (usually seen with at least slight ST elevation). The low voltage is non-specific; although a pericardial syndrome is not excluded, diffuse ST elevations (due to pericarditis) or electrical alternans (a specific but not sensitive sign of tamponade when present with sinus tachycardia) are not seen. Myxedema is classically associated with sinus bradycardia and low voltage complexes ("low and slow"). The QTc interval here--0.45 sec--is slightly prolonged. This may be due to the wide QRS, but also raises question of concomitant hypocalcemia in conjunction with hyperkalemia due to chronic renal failure.
66-yr-old male: what evidence (subtle) is there that he has chronic renal failure?
The tracing shows sinus rhythm with a prolonged PR interval, intraventricular conduction defect (IVCD), left atrial abnormality (LAA), probably underlying left ventricular hypertrophy (LVH) and probably underlying anteroseptal myocardial infraction (MI). Of note is the subtle T wave peaking in V5, with a slightly long QT (.450 ms). The former is consistent with hyperkalemia (5.9 mEq/L); the latter with possible hypocalcemia (level not available). Sometimes hyperkalemia produces only subtle T wave peaking, without absolutely tall T waves, as is the case here.
40-year-old female with life-threatening problem.
Hyperkalemia (associated with chronic renal failure). The K+ one hour before this ECG was 7.4 mEq/L, and likely lower at time of this tracing. Note the subtle T wave peaking ("tenting") here best seen in the lateral chest leads. Although the T waves aren't very tall, they are very narrow-based. Also present is borderline low limb lead voltage and probable left atrial abnormality.
 
31-year-old woman with ECG simulating ischemia. What metabolic abnormality should you suspect?
Hypokalemia. K+ here was 2.2mEq/L. Very severe hypokalemia can cause ST depressions and T-wave inversions as seen here, mimicking ischemia. But also note large U wave (positive deflection after ST-T and before P).
63-year-old man with history of syncope. What is the rhythm?
Marked junctional bradycardia (about 25 beats/min). The rate is regular, without P waves and a flat baseline between narrow QRS complexes. The patient has a history of coronary artery disease with non-diagnostic inferior Q waves noted here. The potassium was slightly elevated at 5.5 mEq/L, and the patient was on atenolol, with possible underlying sick sinus syndrome to account for this rhythm disturbance. (The combination of low voltage and bradyarrhythmia should also raise the possibility of hypothyroidism.) Symptomatic bradycardia of this type persisting after medication adjustments and normalization of serum potassium is an indication for a permanent pacemaker.
This ECG is most likely to be associated with what arrhythmia?    a)   Monomorphic ventricular tachycardia associated with myocardial infarction b)   Ventricular fibrillation associated with transmural ischemia c)   Torsade de pointes-type polymorphic ventricular tachcyardia associated with long QT(U) d)   Paroxysmal atrial fibrillation e)   Ventricular fibrillation due to digitalis toxicity
c) Torsades  The patient had been resuscitated from a cardiac arrest, and this post-arrest ECG revealed marked QT(U) prolongation. The long QT was multifactorial here, associated with tricyclic use, anorexia nervosa, and probable hypokalemia. There are T-U waves best seen in V3 and V4. Echocardiogram showed normal ventricular function and size, despite prominent voltage in this thin adult female.
This ECG shows evidence consistent with all of the following, except:    a)   Left ventricular hypertrophy b)   Left anterior fascicular block c)   Hyperkalemia d)   Right bundle branch block e)   Possible prior anteroseptal myocardial infarction
Peaked ("tented") T waves are seen in concert with left ventricular hypertrophy (LVH), left anterior fascicular block, and very slow R-wave progression V1-V3, the latter finding raising the possibility of prior anteroseptal myocardial infarction (MI). The combination of ECG signs of hyperkalemia and LVH is strongly suggestive of underlying renal insufficiency (with hypertension to account for the LVH). The K+ here was 6.8 meq/L.
This ECG is most consistent with which diagnosis?     a)   Acute anterior myocardial infarction b)   Accelerated idioventricular rhythm c)   Marked hyperkalemia d)   Marked hypercalcemia e)   Systemic hypothermia
c) Marked hyperkalemia The K+ was 10.5 mEq/L in a patient with renal failure. Note the wide QRS complexes with no evident P waves. CK was normal and ST elevations were likely due to hyperkalemia. (Note: The apparent spike after the second QRS complex is an artifact.)
Which clinical syndrome best fits the ECG in this 60-year-old male in the Intensive Care Unit?    a)   Acute anterior myocardial infarction b)   Primary hyperparathyroidism c)   Acute pulmonary embolism d)   Acute renal failure e)   Syndrome of inappropriate ADH (SIADH)
d) Acute renal failure The serum potassium was 8.9 mEq/L. Note the wide QRS with tall ("peaked" or "tented") T waves in V2-V4. The PR interval is prolonged. Left atrial abnormality is also present. (Borderline right axis deviation is present which is non-diagnostic and could be due to incipient left posterior fascicular block associated with hyperkalemia or to other factors such as chronic obstructive pulmonary disease).
Young adult Asian male with Graves' disease and weakness. The ECG is most consistent with which of the following:     a)   Hypokalemia b)   Hyperkalemia c)   Hypocalcemia d)   Hypercalcemia e)   Subendocardial ischemia
a) The patient's serum potassium was 1.9 mEq/L at about the time this ECG was recorded. Resting sinus tachycardia with PR prolongation is present. The P waves in lead II are superimposed on the TU waves. Marked prolongation of repolarization is present with very large U waves (V3-V4, especially). Hyperthyroidism may be associated with hypokalemic periodic paralysis, especially among Asians.
The following ECG with sinus rhythm from a severely-ill 67-year-old woman is most consistent with which diagnosis?    a)   Mitral stenosis b)   Renal failure c)   Myxedema d)   Subarachnoid hemorrhage e)   Acute anterior myocardial infarction
b) The findings of left ventricular hypertrophy (LVH) and tall, narrow (tented) T waves are most consistent with chronic renal failure causing hypertension and hyperkalemia, respectively. Lab studies obtained 30 min after this ECG showed a BUN 42 mg/dl, creatinine 8.0mg/dl, serum potassium 6.4 mEq/L. Severe mitral stenosis classically produces signs of right ventricular hypertrophy and marked left atrial abnormality (or atrial fibrillation). Myxedema (severe hypothyroidism) is associated with low voltage QRS complexes and sinus bradycardia. Subarachnoid hemorrhage may be associated with deep T wave inversions or tall positive T waves, which are usually very broad with a long Q-T, not narrow-based ("CVA T wave pattern"). Finally, the hyperacute T waves of anterior myocardial infarction (MI) are usually also broader, often with a high take-off of the ST segment or J point.
A middle-aged woman with recent onset palpitations. This arrhythmia is most consistent with which endocrine disorder?    a)   Hyperthyroidism b)   Hypothyroidism c)   Hyperparathyroidism d)   Addison's disease e)   Cushing's disease
a) The rhythm is atrial fibrillation with a rapid ventricular response. Left ventricular hypertrophy (LVH) by voltage is noted. The patient was markedly hyperthyroid. An estimated 5-15% of patients with hyperthyroidism (especially older ones) will develop atrial fibrillation. Conversely, a low TSH level was reported in about 5% of patients with recent onset atrial fibrillation (or flutter) Hypothyroidism as well as Addison's disease may be associated with low QRS voltage, non-specific ST-T changes, and sinus bradycardia.Hyperparathyroidism may produce signs of hypercalcemia. Cushing's disease may be associated with LVH (hypertension) and a long Q-T(U) (hypokalemia).
What is the most likely clinical scenario in this patient?     a)   Primary hyperaldosteronism b)   SIADH c)   Metabolic alkalosis d)   Renal failure e)   Hypercalcemia
d) The ECG shows subtle but diagnostic signs of mild-moderate hyperkalemia (6.8mEq/L in this case of renal failure). Specifically, there are narrow-based ("peaked" or "tented") T waves best seen here in leads II and V4-V6. The T waves are not particularly tall, but have a distinctive appearance. Peaking of the T waves is usually the earliest sign of hyperkalemia on the ECG, followed by widening of the QRS and PR prolongation and eventually loss of P waves. Ultimately, a "sine-wave" pattern occurs, preceding ventricular asystole. In some cases, the T waves are quite tall and unmistakable. In other cases, like this one, one needs to pay attention to the narrowness and relative prominence of the T waves. Primary hyperaldosteronism leads to hypokalemia, associated with flat T waves and prominent U waves. Similar changes may occur with metabolic alkalosis which is often associated with hypokalemia.  Syndrome of inappropriate ADH Secretion (SIADH), with hyponatremia, does not by itself produce characteristic changes on the ECG.  Hypercalemia may lead to shortening of the QT interval, due to abbreviation of the ST segment. The present ECG also shows some non-specific J point sagging and left atrial abnormality (prominent biphasic P wave in V1). The latter finding is not uncommon in patients with chronic renal failure who often have hypertension.
 
64-year-old male with a serious metabolic problem:  
The left anterior descending (LAD). The patient actually had multivessel disease. The ECG shows an extensive acute/evolving antero-lateral myocardial infarction (MI) pattern. The rhythm is borderline sinus tachycardia with a single premature atrial complex (PAC)-(4th beat). Note also low limb lead voltage and probable left atrial abnormality (LAA). Left ventriculography showed diffuse hypokinesis as well as akinesis of the antero-lateral and apical walls, with an ejection fraction of 33%.
Left anterior oblique caudal angiographic projection demonstrating total occlusion of the proximal left anterior descending artery (open arrow). Also noted is ramus intermedius disease (closed arrow) as well as circumflex system disease (small arrows).
Transthoracic apical four-chamber echocardiographic image demonstrating antero-septal and apical aneurysmal dilation (arrows) of the left ventricle (LV) post-LAD infarct. LA = left atrium; RV = right ventricle; RA = right atrium.
Dual isotope stress (top rows) and rest (bottom rows) images demonstrating fixed distal anteroseptal (straight white arrows) and infero-basal (angled arrows) perfusion defects. Note both defects are seen in multiple views. Short axis (SA) images are arranged from apex to base; vertical long axis (VLA) images are arranged from septum to lateral wall; and horizontal long axis (HLA) images are arranged from inferior to anterior.
53-yr-old woman. What service is she on?  
Coronary Care Unit.  ST elevations in I and aVL with probable reciprocal ST depressions inferiorly consistent with acute lateral ischemia/MI. Remember that ST elevations like this are never reciprocal but indicate the primary region of ischemia, c/w diagonal or circumflex lesion, confirmed at catheterization in this case of lateral transmural ischemia.
The ECG findings here are most consistent with which life-threatening serum laboratory abnormality? (Note: there is 60 Hz electrical interference in some of the leads here, esp. V1-V2.)    a)   Na+ 110mEq/L b)   K+ 8.5mEq/L c)   K+ 2.0mEq/L d)   Glucose 20mg/dl e)   Elevated CK-MB (20%)
b) The ECG findings are diagnostic of severe hyperkalemia. The two key features are a very wide QRS complex (intraventricular conduction delay) and no evident P waves so that the underlying atrial mechanism cannot be determined. (The bizarre QRS morphology seen here with wide terminal R waves in V1 and V2 and very wide S waves in other leads is also not uncommon in this context; although a variety of other wide complex morphologies are also seen with severe hyperkalemia.  Of further note, ST elevations localized in leads V1-V2, as seen here, are also sometimes observed with severe hyperkalemia in the absence of acute ischemia/myocardial infarction (MI), although the latter cannot be excluded by the ECG alone. CK-MB and troponin assays were negative in this case. The patient had severe hyperkalemia with combined renal and hepatic failure (hepato-renal syndrome).
ECG from a 66-yr-old man with alcoholic cirrhosis and hemodynamic collapse. ECG two weeks prior to this admission showed normal intervals. The present ECG is most consistent with which critical serum laboratory abnormality? (Note: The wavy baseline here is probably due to movement artifact in this critically-ill patient and makes identification of the P waves difficult.)    a)   K+ 8.2 mEq/L b)   Ca++ 15.2 mg/dL c)   TSH >100 microU/ml (markedly increased) d)   Na+ 110 mEq/L e)   CK 3500 IU/L with 15% (increased) MB fraction
a)   K+ 8.2 mEq/L b)   Ca++ 15.2 mg/dL c)   TSH >100 microU/ml (markedly increased) d)   Na+ 110 mEq/L e)   CK 3500 IU/L with 15% (increased) MB fraction
a) The ECG is very suggestive of severe hyperkalemia with the combination of symmetrically peaked T waves (V2-V4) with a wide QRS (left bundle branch block pattern here) and a long PR interval (about 0.26 sec—see lead V2).The wide QRS with severe hyperkalemia can resemble a left or right bundle branch block pattern or often shows more non-specific type of intraventricular conduction disturbance. The effects depend to some extent on any underlying conduction disturbances.Hypercalcemia produces shortening of the QT interval, which is actually prolonged here in the context of the very wide QRS. (The patient’s calcium was low at 7.6 mg/dL with a very low albumin at 1.6 gm/dL.) Severe hypothyroidism (elevated TSH) may produce sinus bradycardia and low QRS voltage. Hyponatremia does not produce distinct ECG findings. The prominent T waves raise consideration of hyperacute repolarization changes due to acute ischemia. However, the ST segments are not elevated and the T waves have a relatively narrow-based and symmetric shape very suggestive of hyperkalemia. Total CK level here was normal.The patient had ascites and renal failure with GI bleeding and probable hepatorenal syndrome, accounting for the severe hyperkalemia.
This ECG is consistent with all of the following EXCEPT?    a)   Digitalis effect  b)   Hypocalcemia  c)   Risk of torsade de pointes  d)   Amiodarone effect  e)   Sotalol effect
The answer is a.  This is NOT dig effect. The ECG shows a very long QT/QTc=0.56 sec/ 0.60 sec. The patient has renal failure with a serum calcium of 7.2 (normal range 8.4-10.2 mg/dL); a serum albumin level of 2.5 (normal range 3.4-4.8 g/dL); low normal serum potassium of 3.6 (normal range 3.5-5.0 mEq/L) after therapy for hyperkalemia, and low normal serum magnesium 1.7 mg/dL (normal 1.6-2.6 mg/dL) about the time this ECG was obtained.Sotalol and amiodarone may also prolong the QT interval, along with a wide range of other medications. Ventricular repolarization prolongation (QT-U) is the major risk factor for torsade de pointes type of ventricular tachycardia. Digitalis, in contrast, shortens ventricular repolarization (QT interval), often producing “scooping” of the ST-T complex.
This ECG is from an elderly woman and is most consistent with which one of the following severe abnormalities?   a)   Hypercalcemia b)   Hyperkalemia c)   Hypernatremia d)   Hypokalemia e)   Hypocalcemia
This ECG shows an extremely wide complex rhythm (QRS duration 0.28 second) at a rate of 70 bpm. This pattern is approaching that of a so-called “sine-wave” morphology indicative of severe hyperkalemia. Atrial activity is not apparent. The patient’s serum potassium level done about ½ hr before was 6.8 mEq/L (and was likely higher at the time of this ECG). She had end-stage renal disease with a BUN of 78 mg/dL and a serum creatinine of 5.2 mg/dL. Emergency therapy of hyperkalemia is required based on this ECG finding.
Young adult male is found unresponsive. The ECG is most consistent with one of the following diagnoses?     a)   Systemic hypothermia b)   Hypercalcemia c)   Tricyclic antidepressant overdose d)   Hypoglycemia e)   Myxedema
This ECG shows classic signs of systemic hypothermia with prominent sinus bradycardia (about 45 bpm) with marked J (Osborn) waves (see, e.g., leads V4 and V5, simulating a bundle branch block or ischemic pattern). There are also prominent T wave inversions and QT prolongation.The patient, with a history of polysubstance abuse, was found unresponsive, last seen the day before. His rectal temperature was 80.0 degrees F. Following rewarming, his ECG completely normalized.  Myxedema coma is classically associated with sinus bradycardia and low voltage. (J waves have been reported with hypercalcemia but giant deflections of the type seen in this case are not a feature of hypercalcemia which may be associated with right precordial lead ST elevation (e.g., V1-V2). J waves have also been reported in subarachnoid hemorrhage, which may also be accompanied by hypothermia.In marked cases, the distinctive, convex J wave (“hypothermic hump”) appears to be an added deflection at the end of the QRS and beginning of the ST segment. The waveform is related pathophysiologically to altered transmural action potential features with hypothermia. Patients with this pattern are at increased risk of ventricular fibrillation which may occur during rewarming.In contrast with Brugada-type patterns which are associated with “coved” (downsloping) or “saddle-back” right precordial J point/ST elevations, the hypothermic pattern is more diffuse.
This ECG is consistent with which of the following markedly abnormal laboratory findings?     a)   Cl- =115 mEq/L b)   Digoxin level >3.5 ng/mL c)   TSH (thyroid stimulating hormone) <0.01 uIU/mL (normal range: 0.30-.3.0); free thyroxine level =5.4 ng/dL (normal range: (0.93-1.7) d)   Na+ = 125 mEq/L e)   K+ = 2.4 mEq/L
This ECG shows sinus bradycardia (52 bpm) with T wave flattening and very prominent U waves, best seen in leads V2-V3, most consistent with severe hypokalemia and/or certain types of drug effect/toxicity. The patient had marked hypokalemia.  A low TSH with an elevated T4 level would be consistent with severe hyperthyroidism, which is associated with resting sinus tachycardia (or atrial fibrillation).  Digoxin toxicity may be associated with sinus bradycardia but the QT interval would be relatively short. Hyponatremia and hyperchloremia are not associated with specific ECG findings.  A long QT-U syndrome, hereditary or acquired, as present here, is the key ECG marker of risk for syncope or sudden cardiac death due to torsade(s) de pointes type ventricular tachycardia.
47 year old woman with a history of breast cancer and a life-threatening problem. Echocardiogram showed normal ventricular function and no pericardial effusion. This ECG is most consistent with which ONE of the following diagnoses?     a)   Hypercalcemia b)   Hyperkalemia c)   Digitalis toxicity d)   Tricyclic antidepressant overdose e)   Systemic hypothermia
The clinical diagnosis was hypercalcemia (Ca++ 15.8 mg/dL) due to metastatic breast cancer. The ECG shows resting sinus tachycardia with borderline left axis deviation. There is also borderline low limb lead voltage. The key finding is the very short <300-320 msec) QT interval (QT=280 msec) such that the T wave appears to take off dirrectly from the J point with marked abbreviation of the ST segment. This finding correlates with shortening of ventricular action potential phase 2 duration. Elevation of the J point in leads V1-V2/V3 is sometimes also seen with hypercalcemia, simulating ischemia.The lower limits of the absolute QT and rate-corrected QT (QTc) intervals are not well defined, and relative shortening occurs physiologically at higher sinus rates. The two major pathologic causes of an acquired relatively short QT are hypercalcemia and digitalis glycoside effect. The latter is excluded here by the absence of characteristic &quot;scooped&quot; ST depressions. A hereditary form of short QT syndrome has been described recently in families with a history of sudden cardiac death.
 
46 year-old man is found unconscious. What is the rhythm?    a)   Polymorphic (torsade-type) ventricular tachycardia b)   Atrial fibrillation with aberration c)   Sinus tachycardia with intermittent runs of monomorphic ventricular tachycardia d)   Multifocal atrial tachycardia with aberration e)   Sinus tachycardia with intermittent right bundle branch block
This ECG shows a relatively regular, wide complex polymorphic ventricular tachycardia, rate 150 bpm, that is most likely a type of torsade(s) de pointes. See leads V4-V6 for classic “turning of the points” morphology. Intermittent ventricular fusion beats may be present. Causes of this life-threatening arrhythmia include a variety of drugs and specific electrolyte abnormalities (especially hypokalemia and hypomagnesemia) associated with long QT (U) intervals, as well as hereditary long QT syndromes due to “channelopathies.”Admission laboratory testing revealed a serum potassium of 2.2 (normal range 3.5-5.3 mEq/L), sodium of 143 (range 135-148 mEq/L), bicarbonate of 16 (normal 21-30mEq/L), phosphate 1.0 (range 2.7-4.5 mg/dL) and magnesium 2.5 (range 1.6-2.6 mg/dL.)
30 year-old man admitted with coma. This ECG is most consistent with which of the following abnormalities?     a)   Tricyclic antidepressant overdose b)   Myxedema c)   Systemic hypothermia d)   Brugada pattern e)   Hypokalemia
This ECG shows classic signs of systemic hypothermia with sinus bradycardia (at about 52 beats/min), prominent J (Osborn) waves and QT prolongation (540 msec). The patient had a body temperature of 26.4 degrees C (79.5 degrees F). He also had hyperkalemia (serum K+ 6.9 mEq/L that may account for the prominent T waves superimposed on the hypothermia pattern and probably on an underlying physiologic early repolarization variant in this young adult man.
What abnormality is suggested by the ECG in this 71 year old man?    a)   Hyperkalemia b)   Digitalis toxicity c)   Systemic hypothermia d)   Hypercalcemia e)   Tricyclic antidepressant overdose
This ECG shows sinus rhythm at a rate of about 70 bpm (there is sinus arrhythmia with a short sinus pause at the beginning). Of particular note is the shortened Q-T interval (about 280 msec) and markedly abbreviated ST segment consistent with hypercalcemia. Subtle “doming” of the ST-T complex in leads V1-V3 with slight J point/ST elevations in the right chest leads is also sometimes seen in this setting. The patient’s serum calcium at the time of this ECG was very elevated at 14.8 mg/dL (normal range 8.4-10.2). After extensive evaluation, a diagnosis of primary hyperparathyroidism was made.
47 year old woman with a history of breast cancer and a life-threatening problem. Echocardiogram showed normal ventricular function and no pericardial effusion. This ECG is most consistent with which one of the following diagnoses?     a)   Acute subarachnoid bleed b)   Acute pulmonary embolism c)   Acute anterior myocardial infarction (MI) d)   Acute pancreatitis e)   Hypercalcemia
The ECG shows a number of salient findings including: sinus tachycardia, long QT/QTc (.34 sec/.46), low limb lead and lateral QRS voltages, along with non-diagnostic slow R wave progression V1-V3 and non-specific ST-T changes. The only diagnosis listed above that accounts for the major findings is acute pancreatitis. The long QT (ST segment component is stretched out) is due to hypocalcemia (Ca++ 7.2 mg/dL with serum albumin of 2.7 g/dL). Sinus tachycardia is due to neuroautonomic activation in this context and the low voltage is consistent with ascites/anasarca.
A 43 year-old man is found unresponsive. What is the most likely diagnosis?    a)   Hyponatremia b)   Brugada pattern c)   Tricyclic antidepressant overdose d)   Systemic hypothermia e)   Myxedema
The rhythm is sinus bradycardia at a rate of about 46 bpm with prominent “J” (Osborn) waves in leads V4-V6 and marked QT prolongation (620 msec). The baseline noise in this context is consistent with shiver-related artifact. This combination of findings is virtually pathognomonic of systemic hypothermia. (The precordial voltage is borderline prominent without definite evidence of left ventricular hypertrophy.)This patient was a homeless man with no known medical history who was found unresponsive with a body temperature of 74.6 degrees F during an extremely cold spell. Laboratory findings revealed a low serum potassium level of 3.2 (normal range 3.5-5.0 mEq/L) that may have contributed to the repolarization prolongation, low serum calcium of 7.3 (normal range 8.4-10.2 mg/dL) proportional to the low serum albumin of 2.7 g/dL, and low normal serum magnesium 1.7 (range 1.6-2.6 mg/dL). A transthoracic echocardiogram showed a normal left ventricular ejection fraction of 60 % and symmetric, diastolic left ventricular wall thickness (septum and posterior wall) at the upper limits of normal (11 mm)
Predict the future!
Patient developed torsade de pointes subsequently. Note marked QT prolongation (QT around 580 msec) in patient on erythromycin and haloperidol.
Why is this young cocaine abuser in the emergency department ?  
He is having an evolving anteroseptal myocardial infarction secondary to cocaine. There are Q waves in V2-V3 with ST segment elevation in V2-V5 associated with T-wave inversion. Also noted are biphasic T-waves in the inferior leads. These multiple abnormalities suggest occlusion of a large left anterior descending (LAD) artery that wraps around the apex of the heart (or multivessel coronary artery disease). Left anterior oblique caudal angiographic projection demonstrating total occlusion of the mid-LAD (open arrow) distal to the takeoff of a large first diagonal branch. Also noted is a 90% tubular stenosis of the first obtuse marginal branch (closed arrow).
A 77 year old man had a post-operative ECG following left lower lobectomy for lung cancer. Prior ECG was normal. He had no chest discomfort or other cardiac symptoms. He was on a continuous epidural infusion of bupivacaine. Echocardiogram showed normal ventricular wall motion and serial CK-MB enzyme tests were normal. The ECG is most consistent with which ONE of the following diagnoses?     a)   Acute pericarditis b)   Acute anterior ST elevation ST elevation MI c)   Acute right ventricular ST elevation MI d)   Brugada-type pattern, drug-induced e)   Benign early repolarization
The ECG shows sinus rhythm with unusual ST segment elevations/T wave inversions (“coved” pattern) in V1 and V2. This morphology and localization is strongly suggestive of a Brugada-type pattern, which may be induced by certain drugs, especially sodium channel blocking agents, in susceptible individuals. The negative enzymes, normal echocardiogram and normalization of the ECG off the epicardial bupivacaine (a reported cause of this finding) are all consistent with the diagnosis drug-induced Brugada pattern. Failure to recognize this finding could result in inappropriate therapy for ST elevation MI or to development of life-threatening ventricular arrhythmias related to the Brugada pathophysiology.

Contenu connexe

Tendances

ECG Quiz
ECG QuizECG Quiz
ECG Quiztmit2
 
Electrocardiogram in myocardial infarction
Electrocardiogram in myocardial infarction Electrocardiogram in myocardial infarction
Electrocardiogram in myocardial infarction Ramachandra Barik
 
Electrolyte and metabolic ECG abnormalities
Electrolyte and metabolic ECG abnormalitiesElectrolyte and metabolic ECG abnormalities
Electrolyte and metabolic ECG abnormalitiesAby Thankachan
 
ECG changes in myocardial Ischemia and injury
ECG changes in myocardial Ischemia and injuryECG changes in myocardial Ischemia and injury
ECG changes in myocardial Ischemia and injurySarah Sreekanth
 
ECG LIMB LEAD REVERSAL
ECG LIMB LEAD REVERSALECG LIMB LEAD REVERSAL
ECG LIMB LEAD REVERSALPraveen Nagula
 
Electrolytes imbalance and ECG changes presentation 2005
Electrolytes imbalance and ECG changes presentation 2005Electrolytes imbalance and ECG changes presentation 2005
Electrolytes imbalance and ECG changes presentation 2005Idrees Muhammad
 
Interpretation of normal 12 leads electrocardiogram &amp; some
Interpretation of normal 12 leads electrocardiogram &amp; someInterpretation of normal 12 leads electrocardiogram &amp; some
Interpretation of normal 12 leads electrocardiogram &amp; someHarihar Adhikari
 
Ecg vtach brugada &amp; vereckei algorithm mec elino
Ecg vtach brugada &amp; vereckei algorithm   mec elinoEcg vtach brugada &amp; vereckei algorithm   mec elino
Ecg vtach brugada &amp; vereckei algorithm mec elinoMeccar Moniem Elino
 
CHANNELOPATHIES - DR RAMDHAN.ppt
CHANNELOPATHIES - DR RAMDHAN.pptCHANNELOPATHIES - DR RAMDHAN.ppt
CHANNELOPATHIES - DR RAMDHAN.pptPDT DM CARDIOLOGY
 
Left Bundle Branch Block (LBBB)
Left Bundle Branch Block (LBBB)Left Bundle Branch Block (LBBB)
Left Bundle Branch Block (LBBB)Kerolus Shehata
 
Localisation of Myocardial Infarction
Localisation of Myocardial InfarctionLocalisation of Myocardial Infarction
Localisation of Myocardial Infarctionakifab93
 
Approach to a case of narrow complex tachycardia
Approach to a case of narrow complex tachycardiaApproach to a case of narrow complex tachycardia
Approach to a case of narrow complex tachycardiaPraveen Nagula
 
STEMI equivalents- ECG update
STEMI equivalents- ECG updateSTEMI equivalents- ECG update
STEMI equivalents- ECG updateMagesh Vadivelu
 
Vt vs svt ab copy
Vt vs svt ab   copyVt vs svt ab   copy
Vt vs svt ab copyGinu George
 

Tendances (20)

ECG Quiz
ECG QuizECG Quiz
ECG Quiz
 
Electrocardiogram in myocardial infarction
Electrocardiogram in myocardial infarction Electrocardiogram in myocardial infarction
Electrocardiogram in myocardial infarction
 
Electrolyte and metabolic ECG abnormalities
Electrolyte and metabolic ECG abnormalitiesElectrolyte and metabolic ECG abnormalities
Electrolyte and metabolic ECG abnormalities
 
ECG changes in myocardial Ischemia and injury
ECG changes in myocardial Ischemia and injuryECG changes in myocardial Ischemia and injury
ECG changes in myocardial Ischemia and injury
 
ECG LIMB LEAD REVERSAL
ECG LIMB LEAD REVERSALECG LIMB LEAD REVERSAL
ECG LIMB LEAD REVERSAL
 
Electrolytes imbalance and ECG changes presentation 2005
Electrolytes imbalance and ECG changes presentation 2005Electrolytes imbalance and ECG changes presentation 2005
Electrolytes imbalance and ECG changes presentation 2005
 
Infarct localisation
Infarct localisationInfarct localisation
Infarct localisation
 
Interpretation of normal 12 leads electrocardiogram &amp; some
Interpretation of normal 12 leads electrocardiogram &amp; someInterpretation of normal 12 leads electrocardiogram &amp; some
Interpretation of normal 12 leads electrocardiogram &amp; some
 
Aritmie
AritmieAritmie
Aritmie
 
Stemi equivalents
Stemi equivalentsStemi equivalents
Stemi equivalents
 
Approach to a patient with T wave abnormality in ECG
Approach to a patient with T wave   abnormality in ECGApproach to a patient with T wave   abnormality in ECG
Approach to a patient with T wave abnormality in ECG
 
Ecg vtach brugada &amp; vereckei algorithm mec elino
Ecg vtach brugada &amp; vereckei algorithm   mec elinoEcg vtach brugada &amp; vereckei algorithm   mec elino
Ecg vtach brugada &amp; vereckei algorithm mec elino
 
CHANNELOPATHIES - DR RAMDHAN.ppt
CHANNELOPATHIES - DR RAMDHAN.pptCHANNELOPATHIES - DR RAMDHAN.ppt
CHANNELOPATHIES - DR RAMDHAN.ppt
 
Left Bundle Branch Block (LBBB)
Left Bundle Branch Block (LBBB)Left Bundle Branch Block (LBBB)
Left Bundle Branch Block (LBBB)
 
WIDE QRS TACHYCARDIA
WIDE  QRS TACHYCARDIAWIDE  QRS TACHYCARDIA
WIDE QRS TACHYCARDIA
 
Localisation of Myocardial Infarction
Localisation of Myocardial InfarctionLocalisation of Myocardial Infarction
Localisation of Myocardial Infarction
 
Approach to a case of narrow complex tachycardia
Approach to a case of narrow complex tachycardiaApproach to a case of narrow complex tachycardia
Approach to a case of narrow complex tachycardia
 
STEMI equivalents- ECG update
STEMI equivalents- ECG updateSTEMI equivalents- ECG update
STEMI equivalents- ECG update
 
STEMI Equivalent
STEMI Equivalent STEMI Equivalent
STEMI Equivalent
 
Vt vs svt ab copy
Vt vs svt ab   copyVt vs svt ab   copy
Vt vs svt ab copy
 

En vedette

In and Out of Potassium - Dr Satish Deopujari
In and Out of Potassium - Dr Satish DeopujariIn and Out of Potassium - Dr Satish Deopujari
In and Out of Potassium - Dr Satish DeopujariCreativity Please
 
10% dextrose usb
10% dextrose usb10% dextrose usb
10% dextrose usbmedicturner
 
Water, Sodium Handling and Hyponatraemia
Water, Sodium Handling and HyponatraemiaWater, Sodium Handling and Hyponatraemia
Water, Sodium Handling and HyponatraemiaRichard McCrory
 
Hyponatremia in Clinical Practice
Hyponatremia in Clinical PracticeHyponatremia in Clinical Practice
Hyponatremia in Clinical PracticeRohan_Roxxx
 
Acute Decompensated Heart Failure : What is New ?
Acute Decompensated Heart Failure : What is New ?Acute Decompensated Heart Failure : What is New ?
Acute Decompensated Heart Failure : What is New ?drucsamal
 
Meninges, ventricles and cerebro spinal fluid
Meninges, ventricles and cerebro spinal fluidMeninges, ventricles and cerebro spinal fluid
Meninges, ventricles and cerebro spinal fluidSahroz Khan
 
Meninges the_spinal_cord
Meninges  the_spinal_cordMeninges  the_spinal_cord
Meninges the_spinal_cordPersonal
 
hypoglycemia and electrolyte imbalance in newborn
hypoglycemia and electrolyte imbalance in newbornhypoglycemia and electrolyte imbalance in newborn
hypoglycemia and electrolyte imbalance in newbornRakesh Verma
 
Gastrointestinal secretions 1
Gastrointestinal secretions 1Gastrointestinal secretions 1
Gastrointestinal secretions 1grandsoltan
 
Ecg & electrolytes disturbance
Ecg & electrolytes disturbanceEcg & electrolytes disturbance
Ecg & electrolytes disturbancealialiali99
 
How is gastric juice production regulated
How is gastric juice production regulatedHow is gastric juice production regulated
How is gastric juice production regulatedBubly Atif
 
Life threatening electrolyte abnormalities
Life threatening electrolyte abnormalitiesLife threatening electrolyte abnormalities
Life threatening electrolyte abnormalitiesChew Keng Sheng
 

En vedette (20)

Mellss yr4 anaest icu management
Mellss yr4 anaest icu managementMellss yr4 anaest icu management
Mellss yr4 anaest icu management
 
In and Out of Potassium - Dr Satish Deopujari
In and Out of Potassium - Dr Satish DeopujariIn and Out of Potassium - Dr Satish Deopujari
In and Out of Potassium - Dr Satish Deopujari
 
10% dextrose usb
10% dextrose usb10% dextrose usb
10% dextrose usb
 
Water, Sodium Handling and Hyponatraemia
Water, Sodium Handling and HyponatraemiaWater, Sodium Handling and Hyponatraemia
Water, Sodium Handling and Hyponatraemia
 
Hyponatremia in Clinical Practice
Hyponatremia in Clinical PracticeHyponatremia in Clinical Practice
Hyponatremia in Clinical Practice
 
Acute Decompensated Heart Failure : What is New ?
Acute Decompensated Heart Failure : What is New ?Acute Decompensated Heart Failure : What is New ?
Acute Decompensated Heart Failure : What is New ?
 
Meninges, ventricles and cerebro spinal fluid
Meninges, ventricles and cerebro spinal fluidMeninges, ventricles and cerebro spinal fluid
Meninges, ventricles and cerebro spinal fluid
 
Neonatal ekg
Neonatal ekgNeonatal ekg
Neonatal ekg
 
Meninges the_spinal_cord
Meninges  the_spinal_cordMeninges  the_spinal_cord
Meninges the_spinal_cord
 
Antiarrhythmic drugs
Antiarrhythmic drugsAntiarrhythmic drugs
Antiarrhythmic drugs
 
hypoglycemia and electrolyte imbalance in newborn
hypoglycemia and electrolyte imbalance in newbornhypoglycemia and electrolyte imbalance in newborn
hypoglycemia and electrolyte imbalance in newborn
 
Gastrointestinal secretions 1
Gastrointestinal secretions 1Gastrointestinal secretions 1
Gastrointestinal secretions 1
 
Ecg & electrolytes disturbance
Ecg & electrolytes disturbanceEcg & electrolytes disturbance
Ecg & electrolytes disturbance
 
How is gastric juice production regulated
How is gastric juice production regulatedHow is gastric juice production regulated
How is gastric juice production regulated
 
Neonatal ecg part2
Neonatal ecg part2Neonatal ecg part2
Neonatal ecg part2
 
ECGpedia - ECG Presentation
ECGpedia - ECG PresentationECGpedia - ECG Presentation
ECGpedia - ECG Presentation
 
Stoma
StomaStoma
Stoma
 
Cranial meninges
Cranial meningesCranial meninges
Cranial meninges
 
Life threatening electrolyte abnormalities
Life threatening electrolyte abnormalitiesLife threatening electrolyte abnormalities
Life threatening electrolyte abnormalities
 
Colostomy
ColostomyColostomy
Colostomy
 

Similaire à May 6, 2009

Hypocalcemia induced camel-hump t-wave, tee-pee sign, and bradycardia in a ca...
Hypocalcemia induced camel-hump t-wave, tee-pee sign, and bradycardia in a ca...Hypocalcemia induced camel-hump t-wave, tee-pee sign, and bradycardia in a ca...
Hypocalcemia induced camel-hump t-wave, tee-pee sign, and bradycardia in a ca...YasserMohammedHassan1
 
Ekg Cases 7 15 09 Level 2 Part 1
Ekg Cases 7 15 09 Level 2 Part 1Ekg Cases 7 15 09 Level 2 Part 1
Ekg Cases 7 15 09 Level 2 Part 1Michael LaCombe
 
ECG life in the fastlane.pptx
ECG life in the fastlane.pptxECG life in the fastlane.pptx
ECG life in the fastlane.pptxabdogamala2
 
ECG, rapid review of essential topics
ECG, rapid review of essential topicsECG, rapid review of essential topics
ECG, rapid review of essential topicsMahdi Najafi
 
SUNDAY ACADEMIC MEET-1.pdf
SUNDAY ACADEMIC MEET-1.pdfSUNDAY ACADEMIC MEET-1.pdf
SUNDAY ACADEMIC MEET-1.pdfRajuMehta12
 
Ecg findings in life threatening conditions
Ecg findings in life threatening conditionsEcg findings in life threatening conditions
Ecg findings in life threatening conditionsMEEQAT HOSPITAL
 
Mimicks Masks of Myocardial Infarction.pptx
Mimicks Masks of Myocardial Infarction.pptxMimicks Masks of Myocardial Infarction.pptx
Mimicks Masks of Myocardial Infarction.pptxRajeshPonnada3
 
STEMI Mimic WHAT IS IT AND HOW TO IDENTIFY IT ?
STEMI Mimic WHAT IS IT AND HOW TO IDENTIFY IT ?STEMI Mimic WHAT IS IT AND HOW TO IDENTIFY IT ?
STEMI Mimic WHAT IS IT AND HOW TO IDENTIFY IT ?Haitham Habtar
 
2. investigation of cardiovascular system )2(
2. investigation of cardiovascular system )2(2. investigation of cardiovascular system )2(
2. investigation of cardiovascular system )2(Ahmad Hamadi
 
2. investigation of cardiovascular system )2(
2. investigation of cardiovascular system )2(2. investigation of cardiovascular system )2(
2. investigation of cardiovascular system )2(Ahmad Hamadi
 
ECG (easy explanation)
ECG (easy explanation)ECG (easy explanation)
ECG (easy explanation)Anika Dahal
 
ECG Cap cuu (1).pptx
ECG Cap cuu (1).pptxECG Cap cuu (1).pptx
ECG Cap cuu (1).pptxHngVMinh5
 
ECG interpretation in emergency settings
ECG interpretation in emergency settingsECG interpretation in emergency settings
ECG interpretation in emergency settingsAimanSaleh5
 
ECG Basics with Prof Dr LK Meher.pptx
ECG Basics with Prof Dr LK Meher.pptxECG Basics with Prof Dr LK Meher.pptx
ECG Basics with Prof Dr LK Meher.pptxSubha Deep
 
Syncope and the ECG
Syncope and the ECGSyncope and the ECG
Syncope and the ECGSCGH ED CME
 

Similaire à May 6, 2009 (20)

Hypocalcemia induced camel-hump t-wave, tee-pee sign, and bradycardia in a ca...
Hypocalcemia induced camel-hump t-wave, tee-pee sign, and bradycardia in a ca...Hypocalcemia induced camel-hump t-wave, tee-pee sign, and bradycardia in a ca...
Hypocalcemia induced camel-hump t-wave, tee-pee sign, and bradycardia in a ca...
 
ECG in young
ECG in youngECG in young
ECG in young
 
Ekg Cases 7 15 09 Level 2 Part 1
Ekg Cases 7 15 09 Level 2 Part 1Ekg Cases 7 15 09 Level 2 Part 1
Ekg Cases 7 15 09 Level 2 Part 1
 
ECG life in the fastlane.pptx
ECG life in the fastlane.pptxECG life in the fastlane.pptx
ECG life in the fastlane.pptx
 
ECG, rapid review of essential topics
ECG, rapid review of essential topicsECG, rapid review of essential topics
ECG, rapid review of essential topics
 
SUNDAY ACADEMIC MEET-1.pdf
SUNDAY ACADEMIC MEET-1.pdfSUNDAY ACADEMIC MEET-1.pdf
SUNDAY ACADEMIC MEET-1.pdf
 
ECG CME NURUL.pptx
ECG CME NURUL.pptxECG CME NURUL.pptx
ECG CME NURUL.pptx
 
Principles of cardiology total.samir rafla
Principles of cardiology total.samir raflaPrinciples of cardiology total.samir rafla
Principles of cardiology total.samir rafla
 
Ecg findings in life threatening conditions
Ecg findings in life threatening conditionsEcg findings in life threatening conditions
Ecg findings in life threatening conditions
 
Mimicks Masks of Myocardial Infarction.pptx
Mimicks Masks of Myocardial Infarction.pptxMimicks Masks of Myocardial Infarction.pptx
Mimicks Masks of Myocardial Infarction.pptx
 
STEMI Mimic WHAT IS IT AND HOW TO IDENTIFY IT ?
STEMI Mimic WHAT IS IT AND HOW TO IDENTIFY IT ?STEMI Mimic WHAT IS IT AND HOW TO IDENTIFY IT ?
STEMI Mimic WHAT IS IT AND HOW TO IDENTIFY IT ?
 
2. investigation of cardiovascular system )2(
2. investigation of cardiovascular system )2(2. investigation of cardiovascular system )2(
2. investigation of cardiovascular system )2(
 
2. investigation of cardiovascular system )2(
2. investigation of cardiovascular system )2(2. investigation of cardiovascular system )2(
2. investigation of cardiovascular system )2(
 
ECG (easy explanation)
ECG (easy explanation)ECG (easy explanation)
ECG (easy explanation)
 
ECG Cap cuu (1).pptx
ECG Cap cuu (1).pptxECG Cap cuu (1).pptx
ECG Cap cuu (1).pptx
 
ECG interpretation in emergency settings
ECG interpretation in emergency settingsECG interpretation in emergency settings
ECG interpretation in emergency settings
 
ECG Basics with Prof Dr LK Meher.pptx
ECG Basics with Prof Dr LK Meher.pptxECG Basics with Prof Dr LK Meher.pptx
ECG Basics with Prof Dr LK Meher.pptx
 
Ecg ppt
Ecg pptEcg ppt
Ecg ppt
 
Arrhythmias
ArrhythmiasArrhythmias
Arrhythmias
 
Syncope and the ECG
Syncope and the ECGSyncope and the ECG
Syncope and the ECG
 

Plus de Michael LaCombe

Plus de Michael LaCombe (10)

Chest Lungs 3
Chest Lungs 3Chest Lungs 3
Chest Lungs 3
 
Chest Film Part 2
Chest Film Part 2Chest Film Part 2
Chest Film Part 2
 
Chest xrays pneumonias
Chest xrays pneumoniasChest xrays pneumonias
Chest xrays pneumonias
 
Chest Film Part 1
Chest Film Part 1Chest Film Part 1
Chest Film Part 1
 
Arrhythmias July 09
Arrhythmias July 09Arrhythmias July 09
Arrhythmias July 09
 
Ekg Cases 7 8 09 Level 1 Part2
Ekg Cases 7 8 09 Level 1 Part2Ekg Cases 7 8 09 Level 1 Part2
Ekg Cases 7 8 09 Level 1 Part2
 
Ekg Cases Jul09.Level One Part 1
Ekg Cases Jul09.Level One Part 1Ekg Cases Jul09.Level One Part 1
Ekg Cases Jul09.Level One Part 1
 
Acls Fmi 2009
Acls Fmi 2009Acls Fmi 2009
Acls Fmi 2009
 
Four causes of CHF09
Four causes of CHF09Four causes of CHF09
Four causes of CHF09
 
April 8, 09 Ppt.
April 8, 09 Ppt.April 8, 09 Ppt.
April 8, 09 Ppt.
 

Dernier

All Time Service Available Call Girls Marine Drive 📳 9820252231 For 18+ VIP C...
All Time Service Available Call Girls Marine Drive 📳 9820252231 For 18+ VIP C...All Time Service Available Call Girls Marine Drive 📳 9820252231 For 18+ VIP C...
All Time Service Available Call Girls Marine Drive 📳 9820252231 For 18+ VIP C...Arohi Goyal
 
Call Girls Varanasi Just Call 9907093804 Top Class Call Girl Service Available
Call Girls Varanasi Just Call 9907093804 Top Class Call Girl Service AvailableCall Girls Varanasi Just Call 9907093804 Top Class Call Girl Service Available
Call Girls Varanasi Just Call 9907093804 Top Class Call Girl Service AvailableDipal Arora
 
Call Girls Bhubaneswar Just Call 9907093804 Top Class Call Girl Service Avail...
Call Girls Bhubaneswar Just Call 9907093804 Top Class Call Girl Service Avail...Call Girls Bhubaneswar Just Call 9907093804 Top Class Call Girl Service Avail...
Call Girls Bhubaneswar Just Call 9907093804 Top Class Call Girl Service Avail...Dipal Arora
 
Call Girls Bareilly Just Call 9907093804 Top Class Call Girl Service Available
Call Girls Bareilly Just Call 9907093804 Top Class Call Girl Service AvailableCall Girls Bareilly Just Call 9907093804 Top Class Call Girl Service Available
Call Girls Bareilly Just Call 9907093804 Top Class Call Girl Service AvailableDipal Arora
 
Call Girls Bangalore Just Call 9907093804 Top Class Call Girl Service Available
Call Girls Bangalore Just Call 9907093804 Top Class Call Girl Service AvailableCall Girls Bangalore Just Call 9907093804 Top Class Call Girl Service Available
Call Girls Bangalore Just Call 9907093804 Top Class Call Girl Service AvailableDipal Arora
 
VIP Russian Call Girls in Varanasi Samaira 8250192130 Independent Escort Serv...
VIP Russian Call Girls in Varanasi Samaira 8250192130 Independent Escort Serv...VIP Russian Call Girls in Varanasi Samaira 8250192130 Independent Escort Serv...
VIP Russian Call Girls in Varanasi Samaira 8250192130 Independent Escort Serv...Neha Kaur
 
Call Girls Ooty Just Call 9907093804 Top Class Call Girl Service Available
Call Girls Ooty Just Call 9907093804 Top Class Call Girl Service AvailableCall Girls Ooty Just Call 9907093804 Top Class Call Girl Service Available
Call Girls Ooty Just Call 9907093804 Top Class Call Girl Service AvailableDipal Arora
 
Bangalore Call Girl Whatsapp Number 100% Complete Your Sexual Needs
Bangalore Call Girl Whatsapp Number 100% Complete Your Sexual NeedsBangalore Call Girl Whatsapp Number 100% Complete Your Sexual Needs
Bangalore Call Girl Whatsapp Number 100% Complete Your Sexual NeedsGfnyt
 
Bangalore Call Girls Nelamangala Number 7001035870 Meetin With Bangalore Esc...
Bangalore Call Girls Nelamangala Number 7001035870  Meetin With Bangalore Esc...Bangalore Call Girls Nelamangala Number 7001035870  Meetin With Bangalore Esc...
Bangalore Call Girls Nelamangala Number 7001035870 Meetin With Bangalore Esc...narwatsonia7
 
Night 7k to 12k Chennai City Center Call Girls 👉👉 7427069034⭐⭐ 100% Genuine E...
Night 7k to 12k Chennai City Center Call Girls 👉👉 7427069034⭐⭐ 100% Genuine E...Night 7k to 12k Chennai City Center Call Girls 👉👉 7427069034⭐⭐ 100% Genuine E...
Night 7k to 12k Chennai City Center Call Girls 👉👉 7427069034⭐⭐ 100% Genuine E...hotbabesbook
 
Best Rate (Guwahati ) Call Girls Guwahati ⟟ 8617370543 ⟟ High Class Call Girl...
Best Rate (Guwahati ) Call Girls Guwahati ⟟ 8617370543 ⟟ High Class Call Girl...Best Rate (Guwahati ) Call Girls Guwahati ⟟ 8617370543 ⟟ High Class Call Girl...
Best Rate (Guwahati ) Call Girls Guwahati ⟟ 8617370543 ⟟ High Class Call Girl...Dipal Arora
 
Book Paid Powai Call Girls Mumbai 𖠋 9930245274 𖠋Low Budget Full Independent H...
Book Paid Powai Call Girls Mumbai 𖠋 9930245274 𖠋Low Budget Full Independent H...Book Paid Powai Call Girls Mumbai 𖠋 9930245274 𖠋Low Budget Full Independent H...
Book Paid Powai Call Girls Mumbai 𖠋 9930245274 𖠋Low Budget Full Independent H...Call Girls in Nagpur High Profile
 
(👑VVIP ISHAAN ) Russian Call Girls Service Navi Mumbai🖕9920874524🖕Independent...
(👑VVIP ISHAAN ) Russian Call Girls Service Navi Mumbai🖕9920874524🖕Independent...(👑VVIP ISHAAN ) Russian Call Girls Service Navi Mumbai🖕9920874524🖕Independent...
(👑VVIP ISHAAN ) Russian Call Girls Service Navi Mumbai🖕9920874524🖕Independent...Taniya Sharma
 
Call Girls Nagpur Just Call 9907093804 Top Class Call Girl Service Available
Call Girls Nagpur Just Call 9907093804 Top Class Call Girl Service AvailableCall Girls Nagpur Just Call 9907093804 Top Class Call Girl Service Available
Call Girls Nagpur Just Call 9907093804 Top Class Call Girl Service AvailableDipal Arora
 
VIP Call Girls Indore Kirti 💚😋 9256729539 🚀 Indore Escorts
VIP Call Girls Indore Kirti 💚😋  9256729539 🚀 Indore EscortsVIP Call Girls Indore Kirti 💚😋  9256729539 🚀 Indore Escorts
VIP Call Girls Indore Kirti 💚😋 9256729539 🚀 Indore Escortsaditipandeya
 
VIP Mumbai Call Girls Hiranandani Gardens Just Call 9920874524 with A/C Room ...
VIP Mumbai Call Girls Hiranandani Gardens Just Call 9920874524 with A/C Room ...VIP Mumbai Call Girls Hiranandani Gardens Just Call 9920874524 with A/C Room ...
VIP Mumbai Call Girls Hiranandani Gardens Just Call 9920874524 with A/C Room ...Garima Khatri
 
Call Girls Jabalpur Just Call 9907093804 Top Class Call Girl Service Available
Call Girls Jabalpur Just Call 9907093804 Top Class Call Girl Service AvailableCall Girls Jabalpur Just Call 9907093804 Top Class Call Girl Service Available
Call Girls Jabalpur Just Call 9907093804 Top Class Call Girl Service AvailableDipal Arora
 
Top Rated Bangalore Call Girls Richmond Circle ⟟ 8250192130 ⟟ Call Me For Gen...
Top Rated Bangalore Call Girls Richmond Circle ⟟ 8250192130 ⟟ Call Me For Gen...Top Rated Bangalore Call Girls Richmond Circle ⟟ 8250192130 ⟟ Call Me For Gen...
Top Rated Bangalore Call Girls Richmond Circle ⟟ 8250192130 ⟟ Call Me For Gen...narwatsonia7
 
VIP Service Call Girls Sindhi Colony 📳 7877925207 For 18+ VIP Call Girl At Th...
VIP Service Call Girls Sindhi Colony 📳 7877925207 For 18+ VIP Call Girl At Th...VIP Service Call Girls Sindhi Colony 📳 7877925207 For 18+ VIP Call Girl At Th...
VIP Service Call Girls Sindhi Colony 📳 7877925207 For 18+ VIP Call Girl At Th...jageshsingh5554
 
Top Rated Hyderabad Call Girls Erragadda ⟟ 6297143586 ⟟ Call Me For Genuine ...
Top Rated  Hyderabad Call Girls Erragadda ⟟ 6297143586 ⟟ Call Me For Genuine ...Top Rated  Hyderabad Call Girls Erragadda ⟟ 6297143586 ⟟ Call Me For Genuine ...
Top Rated Hyderabad Call Girls Erragadda ⟟ 6297143586 ⟟ Call Me For Genuine ...chandars293
 

Dernier (20)

All Time Service Available Call Girls Marine Drive 📳 9820252231 For 18+ VIP C...
All Time Service Available Call Girls Marine Drive 📳 9820252231 For 18+ VIP C...All Time Service Available Call Girls Marine Drive 📳 9820252231 For 18+ VIP C...
All Time Service Available Call Girls Marine Drive 📳 9820252231 For 18+ VIP C...
 
Call Girls Varanasi Just Call 9907093804 Top Class Call Girl Service Available
Call Girls Varanasi Just Call 9907093804 Top Class Call Girl Service AvailableCall Girls Varanasi Just Call 9907093804 Top Class Call Girl Service Available
Call Girls Varanasi Just Call 9907093804 Top Class Call Girl Service Available
 
Call Girls Bhubaneswar Just Call 9907093804 Top Class Call Girl Service Avail...
Call Girls Bhubaneswar Just Call 9907093804 Top Class Call Girl Service Avail...Call Girls Bhubaneswar Just Call 9907093804 Top Class Call Girl Service Avail...
Call Girls Bhubaneswar Just Call 9907093804 Top Class Call Girl Service Avail...
 
Call Girls Bareilly Just Call 9907093804 Top Class Call Girl Service Available
Call Girls Bareilly Just Call 9907093804 Top Class Call Girl Service AvailableCall Girls Bareilly Just Call 9907093804 Top Class Call Girl Service Available
Call Girls Bareilly Just Call 9907093804 Top Class Call Girl Service Available
 
Call Girls Bangalore Just Call 9907093804 Top Class Call Girl Service Available
Call Girls Bangalore Just Call 9907093804 Top Class Call Girl Service AvailableCall Girls Bangalore Just Call 9907093804 Top Class Call Girl Service Available
Call Girls Bangalore Just Call 9907093804 Top Class Call Girl Service Available
 
VIP Russian Call Girls in Varanasi Samaira 8250192130 Independent Escort Serv...
VIP Russian Call Girls in Varanasi Samaira 8250192130 Independent Escort Serv...VIP Russian Call Girls in Varanasi Samaira 8250192130 Independent Escort Serv...
VIP Russian Call Girls in Varanasi Samaira 8250192130 Independent Escort Serv...
 
Call Girls Ooty Just Call 9907093804 Top Class Call Girl Service Available
Call Girls Ooty Just Call 9907093804 Top Class Call Girl Service AvailableCall Girls Ooty Just Call 9907093804 Top Class Call Girl Service Available
Call Girls Ooty Just Call 9907093804 Top Class Call Girl Service Available
 
Bangalore Call Girl Whatsapp Number 100% Complete Your Sexual Needs
Bangalore Call Girl Whatsapp Number 100% Complete Your Sexual NeedsBangalore Call Girl Whatsapp Number 100% Complete Your Sexual Needs
Bangalore Call Girl Whatsapp Number 100% Complete Your Sexual Needs
 
Bangalore Call Girls Nelamangala Number 7001035870 Meetin With Bangalore Esc...
Bangalore Call Girls Nelamangala Number 7001035870  Meetin With Bangalore Esc...Bangalore Call Girls Nelamangala Number 7001035870  Meetin With Bangalore Esc...
Bangalore Call Girls Nelamangala Number 7001035870 Meetin With Bangalore Esc...
 
Night 7k to 12k Chennai City Center Call Girls 👉👉 7427069034⭐⭐ 100% Genuine E...
Night 7k to 12k Chennai City Center Call Girls 👉👉 7427069034⭐⭐ 100% Genuine E...Night 7k to 12k Chennai City Center Call Girls 👉👉 7427069034⭐⭐ 100% Genuine E...
Night 7k to 12k Chennai City Center Call Girls 👉👉 7427069034⭐⭐ 100% Genuine E...
 
Best Rate (Guwahati ) Call Girls Guwahati ⟟ 8617370543 ⟟ High Class Call Girl...
Best Rate (Guwahati ) Call Girls Guwahati ⟟ 8617370543 ⟟ High Class Call Girl...Best Rate (Guwahati ) Call Girls Guwahati ⟟ 8617370543 ⟟ High Class Call Girl...
Best Rate (Guwahati ) Call Girls Guwahati ⟟ 8617370543 ⟟ High Class Call Girl...
 
Book Paid Powai Call Girls Mumbai 𖠋 9930245274 𖠋Low Budget Full Independent H...
Book Paid Powai Call Girls Mumbai 𖠋 9930245274 𖠋Low Budget Full Independent H...Book Paid Powai Call Girls Mumbai 𖠋 9930245274 𖠋Low Budget Full Independent H...
Book Paid Powai Call Girls Mumbai 𖠋 9930245274 𖠋Low Budget Full Independent H...
 
(👑VVIP ISHAAN ) Russian Call Girls Service Navi Mumbai🖕9920874524🖕Independent...
(👑VVIP ISHAAN ) Russian Call Girls Service Navi Mumbai🖕9920874524🖕Independent...(👑VVIP ISHAAN ) Russian Call Girls Service Navi Mumbai🖕9920874524🖕Independent...
(👑VVIP ISHAAN ) Russian Call Girls Service Navi Mumbai🖕9920874524🖕Independent...
 
Call Girls Nagpur Just Call 9907093804 Top Class Call Girl Service Available
Call Girls Nagpur Just Call 9907093804 Top Class Call Girl Service AvailableCall Girls Nagpur Just Call 9907093804 Top Class Call Girl Service Available
Call Girls Nagpur Just Call 9907093804 Top Class Call Girl Service Available
 
VIP Call Girls Indore Kirti 💚😋 9256729539 🚀 Indore Escorts
VIP Call Girls Indore Kirti 💚😋  9256729539 🚀 Indore EscortsVIP Call Girls Indore Kirti 💚😋  9256729539 🚀 Indore Escorts
VIP Call Girls Indore Kirti 💚😋 9256729539 🚀 Indore Escorts
 
VIP Mumbai Call Girls Hiranandani Gardens Just Call 9920874524 with A/C Room ...
VIP Mumbai Call Girls Hiranandani Gardens Just Call 9920874524 with A/C Room ...VIP Mumbai Call Girls Hiranandani Gardens Just Call 9920874524 with A/C Room ...
VIP Mumbai Call Girls Hiranandani Gardens Just Call 9920874524 with A/C Room ...
 
Call Girls Jabalpur Just Call 9907093804 Top Class Call Girl Service Available
Call Girls Jabalpur Just Call 9907093804 Top Class Call Girl Service AvailableCall Girls Jabalpur Just Call 9907093804 Top Class Call Girl Service Available
Call Girls Jabalpur Just Call 9907093804 Top Class Call Girl Service Available
 
Top Rated Bangalore Call Girls Richmond Circle ⟟ 8250192130 ⟟ Call Me For Gen...
Top Rated Bangalore Call Girls Richmond Circle ⟟ 8250192130 ⟟ Call Me For Gen...Top Rated Bangalore Call Girls Richmond Circle ⟟ 8250192130 ⟟ Call Me For Gen...
Top Rated Bangalore Call Girls Richmond Circle ⟟ 8250192130 ⟟ Call Me For Gen...
 
VIP Service Call Girls Sindhi Colony 📳 7877925207 For 18+ VIP Call Girl At Th...
VIP Service Call Girls Sindhi Colony 📳 7877925207 For 18+ VIP Call Girl At Th...VIP Service Call Girls Sindhi Colony 📳 7877925207 For 18+ VIP Call Girl At Th...
VIP Service Call Girls Sindhi Colony 📳 7877925207 For 18+ VIP Call Girl At Th...
 
Top Rated Hyderabad Call Girls Erragadda ⟟ 6297143586 ⟟ Call Me For Genuine ...
Top Rated  Hyderabad Call Girls Erragadda ⟟ 6297143586 ⟟ Call Me For Genuine ...Top Rated  Hyderabad Call Girls Erragadda ⟟ 6297143586 ⟟ Call Me For Genuine ...
Top Rated Hyderabad Call Girls Erragadda ⟟ 6297143586 ⟟ Call Me For Genuine ...
 

May 6, 2009

  • 1. The EKG in: Metabolic and Electrolyte Disorders MDFMR/UNECOM M. LaCombe May 6, 2009
  • 2. The normal EKG waveform
  • 4. The frontal plane (QRS) axis:
  • 5. Elderly female admitted with obtundation. What electrolyte abnormality is strongly suggested?     a)   Hyponatremia b)   Hypernatremia c)   Hyperkalemia d)   Hypokalemia e)   Hypercalcemia
  • 6. Hyperkalemia (K+ = 8.7 mEq/L) secondary to acute renal failure. The ECG shows symmetrically peaked (&quot;tented&quot;) T waves associated with potassium levels in excess of 6 mEq/L. The tracing also shows broad and flattened sinus P waves that may precede frank sino-ventricular conduction seen with severe hyperkalemia (i.e., conduction from the sinus node to the ventricles through specialized inter-nodal tissue without atrial depolarization). This conduction pattern may simulate a junctional rhythm. The narrow QRS complex in this tracing is somewhat atypical for severe hyperkalemia. Milder degrees of hyperkalemia may actually facilitate AV conduction. Note that T wave peaking with hyperkalemia is a relative finding: the absolute magnitude of the T waves cannot be used to rule in or rule out hyperkalemia. Indeed, in some cases of hyperkalemia with left ventricular hypertrophy, the T wave peaking may affect inverted T waves or cause relative normalization. The presence of relatively low voltage and sinus bradycardia in this clinical setting should also raise consideration of hypothyroidism/myxedema, which by itself usually also causes T wave flattening.
  • 7. This is the admitting ECG of a previously healthy 49-year-old man who presented with progressive muscle weakness and constipation. He had no chest pain or dyspnea. The ECG is most consistent with which ONE of the following diagnoses?  a)   Hypokalemia b)   Hyperkalemia c)   Hypocalcemia d)   Hypercalcemia e)   Hypothyroidism
  • 8. The ECG shows sinus rhythm with the following abnormalities a very short ST segment with a consequently short QT interval (about 300 msec here). The differential diagnosis of a short QT (lower limits are not well-defined) is much narrower than that of prolonged QT. The two chief causes of a short QT are hypercalcemia and digoxin therapy (associated with characteristic &quot;scooping&quot; of the ST-T complex). A third and relatively rare cause is hereditary short QT (&quot;channelopathy&quot;-related) that may be associated with ventricular arrhythmia and sudden cardiac arrest. Cardiac arrhythmias, however, are unusual with hypercalcemia; but AV block, sinus arrest, sino-atrial block, ventricular tachycardia, and cardiac arrest have been reported, usually in patients receiving rapid IV injections of calcium. This patient's serum calcium was 16 mg/dl. He was found to have hyperparathyroidism; at surgery, he had successful resection of a parathyroid carcinoma, an ususual cause of hypercalcemia.
  • 9. The patient is an elderly man who presented to the emergency ward with dizziness and new renal failure.
  • 10. Dx: Hyperkalemia (7.6 mEq/L) secondary to renal failure. The ECG demonstrates findings consistent with severe hyperkalemia -- most importantly marked widening of the QRS complex. The QRS complex here shows a left bundle branch block (LBBB) morphology with left axis deviation. However, the QRS duration (about 240 ms) is much wider than seen with an &quot;uncomplicated&quot; LBBB or just with left anterior fascicular block due to intrinsic conduction disease. There is also peaking of the T waves with prolongation of the PR interval and flattening of the P waves. If the hyperkalemia is left untreated, the ECG will progress to a sinusoidal pattern and eventually asystole with subsequent hemodynamic collapse and death.
  • 11. A 38-yr-old woman with weakness. Previous ECG was normal and she was on no medications. What is the most likely diagnosis?     a)   Hypercalcemia b)   Hypernatremia c)   Hypokalemia d)   Hypocalcemia e)   Hyponatremia
  • 12. The ECG shows sinus bradycardia with diffuse T wave flattening or inversions, and markedly prominent U waves. These are best seen in leads V2 and V3, but are essentially invisible in lead aVL. The two most common causes of this finding are 1) Hypokalemia (K+ here was 2.4 mEq/L) and 2) Drugs, especially the class 1A antiarrhythmics (like quinidine, procainamide, disopyramide) and related agents (like the phenothiazines and tricyclics), etc. Patients with hereditary (congenital) long QT syndromes due to &quot;channelopathies&quot; may show a similar finding. This ventricular repolarization prolongation pattern is of great importance because it identifies patients at high risk of torsade de pointes type of polymorphic ventricular tachycardia.
  • 13. This ECG &quot;speaks&quot; major clues about the underlying major medical condition in this 77-year-old woman. What does it say?
  • 14. ECG shows sinus rhythm with left ventricular hypertrophy (LVH), left atrial abnormality and tall peaked T waves best seen in the precordial leads (with non-specific infero-lateral ST depressions). Also present are left axis deviation consistent with left anterior fascicular block and a borderline long QT. Putting it all together, the peaked T waves indicate increased K+ (6.3 mEq/L) with LVH suggesting renal failure associated with hypertension, which was the case here. The long QT with stretchout ST segment phase goes with decreased calcium seen with renal failure as well. ST depressions could be from LVH or primary ischemia, in addition to multiple other causes. So when you see the triad of &quot;tented&quot; T waves (hyperkalemia) with long QT (ST segment prolongation consistent with hypocalcemia) and LVH (hypertension) the patient virtually always has renal failure.
  • 15. A 65-year-old man. What should you do before calling cardiology fellow for a &quot;hyperacute myocardial infarction&quot; here?
  • 16. This is classic hyperkalemia with tall peaked T waves, along with PR prolongation and underlying left ventricular hypertrophy (LVH) (patient has renal disease with hypertension). Note QRS prolongation also seen with moderate-severe hyperkalemia. Potassium here was 9.6mEq/L. Not all tall positive T waves are &quot;hyperacute.&quot; The latter term should be reserved for increased T wave positivity secondary to transmural ischemia. Major clues include narrowness (tenting) of T waves along with other findings noted above.
  • 17. 30-yr-old man, not on medication. What waveform is prominent? What is the likely diagnosis, assuming prior normal ECG? 
  • 18. There are very (!) prominent U waves with Q-T(U) prolongation. He had severe hypokalemia (1.5 mEq/L) due to diarrhea. Calcium was normal, as was magnesium (remember pure hypocalcemia prolongs the ST segment primarily but does not give large U waves.) Obviously, this acquired-type long-QT(U) syndrome puts the subject at risk for torsade de pointes which he fortunately did not have before coming to hospital. The same type ECG may occur with drug-related QT(U) prolongation syndromes and with hereditary long QT(U) syndromes related to &quot;channelopathies,&quot; including the Romano-Ward and Jervell Lange-Nielsen syndromes.
  • 19. Paris
  • 20. 34-year-old male. What's going on here?
  • 21. ECG shows classic findings of acute/hyperacute anterior wall Q wave myocardial infarction (MI), with reciprocal inferior ST depressions. The distribution of changes is consistent with a proximal left anterior descending (LAD) occlusion which was confirmed at cardiac catheterization and treated with angioplasty/stent. Premature atherosclerosis here was associated with multiple risk factors for coronary artery disease:hypertension, hyperlipidemia, family history and tobacco.
  • 22. Minimalist/optimal medicine quiz. If you could do only one lab test, what would it be in this case?
  • 23. Check the calcium (and yes, of course albumin). It was low here at 8.2 mg/dl with minimally low albumin in patient post thyroidectomy. ECG shows QT prolongation. Most important is that QT is long in this case because the ST segment is stretched out. This relates to prolongation of the plateau phase of action potential which is prolonged with hypocalcemia. Ventricular repolarization can also be long because of 1) wide QRS; 2) flat, wide T waves (e.g. quinidine, sotalol, hypokalemia, etc); and/or 3) prominent U waves melding with T wave. The latter two syndromes, representing dispersion of ventricular refractoriness, are most likely to be associated with torsade de pointes.
  • 25. Serum K+. It was 7.7 mEq/L. Note the relatively tall peaked (tented) T waves that are a classic sign of mild-moderate hyperkalemia. With more severe elevations, the QRS will widen, as will the PR, and the P wave amplitude will diminish, resulting in a junctional or idioventricular-type rhythm eventually.
  • 26. 35-yr-old: If you could only obtain one lab test what would it be?
  • 27. Classic (!) ECG for hypercalcemia (13.9 mg/dl here). Note the very abbreviated ST segment such that the T wave looks like it takes off directly from QRS in some leads. High take-off of ST segment in V2/V3 is also sometimes seen in this context. Hypocalcemia is just the opposite: prolonged QT due to long ST segment (corresponding to changes in action potential plateau phase duration).
  • 28. 51-yr-old male; what is the key ECG finding and its differential diagnosis?
  • 29. The most notable finding here is a markedly prolonged QT(U) interval, best seen in the lateral precordial leads. The differential diagnosis of this important repolarization abnormality centers on 1) electrolyte disorders, especially hypokalemia and hypomagnesemia (the K+ here was at most 3.6 mEq/L, and probably lower, with a borderline low Mg++ of 1.6 mEq/L; the serum calcium was normal); and 2) drug effects (e.g., quinidine, sotalol, amiodarone, tricyclic antidepressants,etc). Hereditary long QT syndromes are much rarer. Hypocalcemia by itself stretches out the ST segment but does not produce prominent U waves. The long QT(U) with hypokalemia/hypomagnesemia is associated with an increased risk of torsade de pointes in this setting.
  • 30. This ECG is most consistent with:    a)   Acute anterior myocardial infarction (MI) b)   Pericarditis/tamponade c)   Renal failure d)   Tricyclic antidepressant overdose e)   Myxedema coma
  • 31. c) There is classic evidence of hyperkalemia with tall, narrow T waves, most evident in leads V3-V4, along with some QRS prolongation. These &quot;tented T waves&quot; are different from the broader hyperacute T waves of acute MI (usually seen with at least slight ST elevation). The low voltage is non-specific; although a pericardial syndrome is not excluded, diffuse ST elevations (due to pericarditis) or electrical alternans (a specific but not sensitive sign of tamponade when present with sinus tachycardia) are not seen. Myxedema is classically associated with sinus bradycardia and low voltage complexes (&quot;low and slow&quot;). The QTc interval here--0.45 sec--is slightly prolonged. This may be due to the wide QRS, but also raises question of concomitant hypocalcemia in conjunction with hyperkalemia due to chronic renal failure.
  • 32. 66-yr-old male: what evidence (subtle) is there that he has chronic renal failure?
  • 33. The tracing shows sinus rhythm with a prolonged PR interval, intraventricular conduction defect (IVCD), left atrial abnormality (LAA), probably underlying left ventricular hypertrophy (LVH) and probably underlying anteroseptal myocardial infraction (MI). Of note is the subtle T wave peaking in V5, with a slightly long QT (.450 ms). The former is consistent with hyperkalemia (5.9 mEq/L); the latter with possible hypocalcemia (level not available). Sometimes hyperkalemia produces only subtle T wave peaking, without absolutely tall T waves, as is the case here.
  • 34. 40-year-old female with life-threatening problem.
  • 35. Hyperkalemia (associated with chronic renal failure). The K+ one hour before this ECG was 7.4 mEq/L, and likely lower at time of this tracing. Note the subtle T wave peaking (&quot;tenting&quot;) here best seen in the lateral chest leads. Although the T waves aren't very tall, they are very narrow-based. Also present is borderline low limb lead voltage and probable left atrial abnormality.
  • 36.  
  • 37. 31-year-old woman with ECG simulating ischemia. What metabolic abnormality should you suspect?
  • 38. Hypokalemia. K+ here was 2.2mEq/L. Very severe hypokalemia can cause ST depressions and T-wave inversions as seen here, mimicking ischemia. But also note large U wave (positive deflection after ST-T and before P).
  • 39. 63-year-old man with history of syncope. What is the rhythm?
  • 40. Marked junctional bradycardia (about 25 beats/min). The rate is regular, without P waves and a flat baseline between narrow QRS complexes. The patient has a history of coronary artery disease with non-diagnostic inferior Q waves noted here. The potassium was slightly elevated at 5.5 mEq/L, and the patient was on atenolol, with possible underlying sick sinus syndrome to account for this rhythm disturbance. (The combination of low voltage and bradyarrhythmia should also raise the possibility of hypothyroidism.) Symptomatic bradycardia of this type persisting after medication adjustments and normalization of serum potassium is an indication for a permanent pacemaker.
  • 41. This ECG is most likely to be associated with what arrhythmia?    a)   Monomorphic ventricular tachycardia associated with myocardial infarction b)   Ventricular fibrillation associated with transmural ischemia c)   Torsade de pointes-type polymorphic ventricular tachcyardia associated with long QT(U) d)   Paroxysmal atrial fibrillation e)   Ventricular fibrillation due to digitalis toxicity
  • 42. c) Torsades The patient had been resuscitated from a cardiac arrest, and this post-arrest ECG revealed marked QT(U) prolongation. The long QT was multifactorial here, associated with tricyclic use, anorexia nervosa, and probable hypokalemia. There are T-U waves best seen in V3 and V4. Echocardiogram showed normal ventricular function and size, despite prominent voltage in this thin adult female.
  • 43. This ECG shows evidence consistent with all of the following, except:    a)   Left ventricular hypertrophy b)   Left anterior fascicular block c)   Hyperkalemia d)   Right bundle branch block e)   Possible prior anteroseptal myocardial infarction
  • 44. Peaked (&quot;tented&quot;) T waves are seen in concert with left ventricular hypertrophy (LVH), left anterior fascicular block, and very slow R-wave progression V1-V3, the latter finding raising the possibility of prior anteroseptal myocardial infarction (MI). The combination of ECG signs of hyperkalemia and LVH is strongly suggestive of underlying renal insufficiency (with hypertension to account for the LVH). The K+ here was 6.8 meq/L.
  • 45. This ECG is most consistent with which diagnosis?     a)   Acute anterior myocardial infarction b)   Accelerated idioventricular rhythm c)   Marked hyperkalemia d)   Marked hypercalcemia e)   Systemic hypothermia
  • 46. c) Marked hyperkalemia The K+ was 10.5 mEq/L in a patient with renal failure. Note the wide QRS complexes with no evident P waves. CK was normal and ST elevations were likely due to hyperkalemia. (Note: The apparent spike after the second QRS complex is an artifact.)
  • 47. Which clinical syndrome best fits the ECG in this 60-year-old male in the Intensive Care Unit?    a)   Acute anterior myocardial infarction b)   Primary hyperparathyroidism c)   Acute pulmonary embolism d)   Acute renal failure e)   Syndrome of inappropriate ADH (SIADH)
  • 48. d) Acute renal failure The serum potassium was 8.9 mEq/L. Note the wide QRS with tall (&quot;peaked&quot; or &quot;tented&quot;) T waves in V2-V4. The PR interval is prolonged. Left atrial abnormality is also present. (Borderline right axis deviation is present which is non-diagnostic and could be due to incipient left posterior fascicular block associated with hyperkalemia or to other factors such as chronic obstructive pulmonary disease).
  • 49. Young adult Asian male with Graves' disease and weakness. The ECG is most consistent with which of the following:     a)   Hypokalemia b)   Hyperkalemia c)   Hypocalcemia d)   Hypercalcemia e)   Subendocardial ischemia
  • 50. a) The patient's serum potassium was 1.9 mEq/L at about the time this ECG was recorded. Resting sinus tachycardia with PR prolongation is present. The P waves in lead II are superimposed on the TU waves. Marked prolongation of repolarization is present with very large U waves (V3-V4, especially). Hyperthyroidism may be associated with hypokalemic periodic paralysis, especially among Asians.
  • 51. The following ECG with sinus rhythm from a severely-ill 67-year-old woman is most consistent with which diagnosis?    a)   Mitral stenosis b)   Renal failure c)   Myxedema d)   Subarachnoid hemorrhage e)   Acute anterior myocardial infarction
  • 52. b) The findings of left ventricular hypertrophy (LVH) and tall, narrow (tented) T waves are most consistent with chronic renal failure causing hypertension and hyperkalemia, respectively. Lab studies obtained 30 min after this ECG showed a BUN 42 mg/dl, creatinine 8.0mg/dl, serum potassium 6.4 mEq/L. Severe mitral stenosis classically produces signs of right ventricular hypertrophy and marked left atrial abnormality (or atrial fibrillation). Myxedema (severe hypothyroidism) is associated with low voltage QRS complexes and sinus bradycardia. Subarachnoid hemorrhage may be associated with deep T wave inversions or tall positive T waves, which are usually very broad with a long Q-T, not narrow-based (&quot;CVA T wave pattern&quot;). Finally, the hyperacute T waves of anterior myocardial infarction (MI) are usually also broader, often with a high take-off of the ST segment or J point.
  • 53. A middle-aged woman with recent onset palpitations. This arrhythmia is most consistent with which endocrine disorder?    a)   Hyperthyroidism b)   Hypothyroidism c)   Hyperparathyroidism d)   Addison's disease e)   Cushing's disease
  • 54. a) The rhythm is atrial fibrillation with a rapid ventricular response. Left ventricular hypertrophy (LVH) by voltage is noted. The patient was markedly hyperthyroid. An estimated 5-15% of patients with hyperthyroidism (especially older ones) will develop atrial fibrillation. Conversely, a low TSH level was reported in about 5% of patients with recent onset atrial fibrillation (or flutter) Hypothyroidism as well as Addison's disease may be associated with low QRS voltage, non-specific ST-T changes, and sinus bradycardia.Hyperparathyroidism may produce signs of hypercalcemia. Cushing's disease may be associated with LVH (hypertension) and a long Q-T(U) (hypokalemia).
  • 55. What is the most likely clinical scenario in this patient?     a)   Primary hyperaldosteronism b)   SIADH c)   Metabolic alkalosis d)   Renal failure e)   Hypercalcemia
  • 56. d) The ECG shows subtle but diagnostic signs of mild-moderate hyperkalemia (6.8mEq/L in this case of renal failure). Specifically, there are narrow-based (&quot;peaked&quot; or &quot;tented&quot;) T waves best seen here in leads II and V4-V6. The T waves are not particularly tall, but have a distinctive appearance. Peaking of the T waves is usually the earliest sign of hyperkalemia on the ECG, followed by widening of the QRS and PR prolongation and eventually loss of P waves. Ultimately, a &quot;sine-wave&quot; pattern occurs, preceding ventricular asystole. In some cases, the T waves are quite tall and unmistakable. In other cases, like this one, one needs to pay attention to the narrowness and relative prominence of the T waves. Primary hyperaldosteronism leads to hypokalemia, associated with flat T waves and prominent U waves. Similar changes may occur with metabolic alkalosis which is often associated with hypokalemia. Syndrome of inappropriate ADH Secretion (SIADH), with hyponatremia, does not by itself produce characteristic changes on the ECG. Hypercalemia may lead to shortening of the QT interval, due to abbreviation of the ST segment. The present ECG also shows some non-specific J point sagging and left atrial abnormality (prominent biphasic P wave in V1). The latter finding is not uncommon in patients with chronic renal failure who often have hypertension.
  • 57.  
  • 58. 64-year-old male with a serious metabolic problem:  
  • 59. The left anterior descending (LAD). The patient actually had multivessel disease. The ECG shows an extensive acute/evolving antero-lateral myocardial infarction (MI) pattern. The rhythm is borderline sinus tachycardia with a single premature atrial complex (PAC)-(4th beat). Note also low limb lead voltage and probable left atrial abnormality (LAA). Left ventriculography showed diffuse hypokinesis as well as akinesis of the antero-lateral and apical walls, with an ejection fraction of 33%.
  • 60. Left anterior oblique caudal angiographic projection demonstrating total occlusion of the proximal left anterior descending artery (open arrow). Also noted is ramus intermedius disease (closed arrow) as well as circumflex system disease (small arrows).
  • 61. Transthoracic apical four-chamber echocardiographic image demonstrating antero-septal and apical aneurysmal dilation (arrows) of the left ventricle (LV) post-LAD infarct. LA = left atrium; RV = right ventricle; RA = right atrium.
  • 62. Dual isotope stress (top rows) and rest (bottom rows) images demonstrating fixed distal anteroseptal (straight white arrows) and infero-basal (angled arrows) perfusion defects. Note both defects are seen in multiple views. Short axis (SA) images are arranged from apex to base; vertical long axis (VLA) images are arranged from septum to lateral wall; and horizontal long axis (HLA) images are arranged from inferior to anterior.
  • 63. 53-yr-old woman. What service is she on?  
  • 64. Coronary Care Unit. ST elevations in I and aVL with probable reciprocal ST depressions inferiorly consistent with acute lateral ischemia/MI. Remember that ST elevations like this are never reciprocal but indicate the primary region of ischemia, c/w diagonal or circumflex lesion, confirmed at catheterization in this case of lateral transmural ischemia.
  • 65. The ECG findings here are most consistent with which life-threatening serum laboratory abnormality? (Note: there is 60 Hz electrical interference in some of the leads here, esp. V1-V2.)    a)   Na+ 110mEq/L b)   K+ 8.5mEq/L c)   K+ 2.0mEq/L d)   Glucose 20mg/dl e)   Elevated CK-MB (20%)
  • 66. b) The ECG findings are diagnostic of severe hyperkalemia. The two key features are a very wide QRS complex (intraventricular conduction delay) and no evident P waves so that the underlying atrial mechanism cannot be determined. (The bizarre QRS morphology seen here with wide terminal R waves in V1 and V2 and very wide S waves in other leads is also not uncommon in this context; although a variety of other wide complex morphologies are also seen with severe hyperkalemia. Of further note, ST elevations localized in leads V1-V2, as seen here, are also sometimes observed with severe hyperkalemia in the absence of acute ischemia/myocardial infarction (MI), although the latter cannot be excluded by the ECG alone. CK-MB and troponin assays were negative in this case. The patient had severe hyperkalemia with combined renal and hepatic failure (hepato-renal syndrome).
  • 67. ECG from a 66-yr-old man with alcoholic cirrhosis and hemodynamic collapse. ECG two weeks prior to this admission showed normal intervals. The present ECG is most consistent with which critical serum laboratory abnormality? (Note: The wavy baseline here is probably due to movement artifact in this critically-ill patient and makes identification of the P waves difficult.)    a)   K+ 8.2 mEq/L b)   Ca++ 15.2 mg/dL c)   TSH >100 microU/ml (markedly increased) d)   Na+ 110 mEq/L e)   CK 3500 IU/L with 15% (increased) MB fraction
  • 68. a)   K+ 8.2 mEq/L b)   Ca++ 15.2 mg/dL c)   TSH >100 microU/ml (markedly increased) d)   Na+ 110 mEq/L e)   CK 3500 IU/L with 15% (increased) MB fraction
  • 69. a) The ECG is very suggestive of severe hyperkalemia with the combination of symmetrically peaked T waves (V2-V4) with a wide QRS (left bundle branch block pattern here) and a long PR interval (about 0.26 sec—see lead V2).The wide QRS with severe hyperkalemia can resemble a left or right bundle branch block pattern or often shows more non-specific type of intraventricular conduction disturbance. The effects depend to some extent on any underlying conduction disturbances.Hypercalcemia produces shortening of the QT interval, which is actually prolonged here in the context of the very wide QRS. (The patient’s calcium was low at 7.6 mg/dL with a very low albumin at 1.6 gm/dL.) Severe hypothyroidism (elevated TSH) may produce sinus bradycardia and low QRS voltage. Hyponatremia does not produce distinct ECG findings. The prominent T waves raise consideration of hyperacute repolarization changes due to acute ischemia. However, the ST segments are not elevated and the T waves have a relatively narrow-based and symmetric shape very suggestive of hyperkalemia. Total CK level here was normal.The patient had ascites and renal failure with GI bleeding and probable hepatorenal syndrome, accounting for the severe hyperkalemia.
  • 70. This ECG is consistent with all of the following EXCEPT?    a)   Digitalis effect b)   Hypocalcemia c)   Risk of torsade de pointes d)   Amiodarone effect e)   Sotalol effect
  • 71. The answer is a. This is NOT dig effect. The ECG shows a very long QT/QTc=0.56 sec/ 0.60 sec. The patient has renal failure with a serum calcium of 7.2 (normal range 8.4-10.2 mg/dL); a serum albumin level of 2.5 (normal range 3.4-4.8 g/dL); low normal serum potassium of 3.6 (normal range 3.5-5.0 mEq/L) after therapy for hyperkalemia, and low normal serum magnesium 1.7 mg/dL (normal 1.6-2.6 mg/dL) about the time this ECG was obtained.Sotalol and amiodarone may also prolong the QT interval, along with a wide range of other medications. Ventricular repolarization prolongation (QT-U) is the major risk factor for torsade de pointes type of ventricular tachycardia. Digitalis, in contrast, shortens ventricular repolarization (QT interval), often producing “scooping” of the ST-T complex.
  • 72. This ECG is from an elderly woman and is most consistent with which one of the following severe abnormalities?   a)   Hypercalcemia b)   Hyperkalemia c)   Hypernatremia d)   Hypokalemia e)   Hypocalcemia
  • 73. This ECG shows an extremely wide complex rhythm (QRS duration 0.28 second) at a rate of 70 bpm. This pattern is approaching that of a so-called “sine-wave” morphology indicative of severe hyperkalemia. Atrial activity is not apparent. The patient’s serum potassium level done about ½ hr before was 6.8 mEq/L (and was likely higher at the time of this ECG). She had end-stage renal disease with a BUN of 78 mg/dL and a serum creatinine of 5.2 mg/dL. Emergency therapy of hyperkalemia is required based on this ECG finding.
  • 74. Young adult male is found unresponsive. The ECG is most consistent with one of the following diagnoses?     a)   Systemic hypothermia b)   Hypercalcemia c)   Tricyclic antidepressant overdose d)   Hypoglycemia e)   Myxedema
  • 75. This ECG shows classic signs of systemic hypothermia with prominent sinus bradycardia (about 45 bpm) with marked J (Osborn) waves (see, e.g., leads V4 and V5, simulating a bundle branch block or ischemic pattern). There are also prominent T wave inversions and QT prolongation.The patient, with a history of polysubstance abuse, was found unresponsive, last seen the day before. His rectal temperature was 80.0 degrees F. Following rewarming, his ECG completely normalized. Myxedema coma is classically associated with sinus bradycardia and low voltage. (J waves have been reported with hypercalcemia but giant deflections of the type seen in this case are not a feature of hypercalcemia which may be associated with right precordial lead ST elevation (e.g., V1-V2). J waves have also been reported in subarachnoid hemorrhage, which may also be accompanied by hypothermia.In marked cases, the distinctive, convex J wave (“hypothermic hump”) appears to be an added deflection at the end of the QRS and beginning of the ST segment. The waveform is related pathophysiologically to altered transmural action potential features with hypothermia. Patients with this pattern are at increased risk of ventricular fibrillation which may occur during rewarming.In contrast with Brugada-type patterns which are associated with “coved” (downsloping) or “saddle-back” right precordial J point/ST elevations, the hypothermic pattern is more diffuse.
  • 76. This ECG is consistent with which of the following markedly abnormal laboratory findings?     a)   Cl- =115 mEq/L b)   Digoxin level >3.5 ng/mL c)   TSH (thyroid stimulating hormone) <0.01 uIU/mL (normal range: 0.30-.3.0); free thyroxine level =5.4 ng/dL (normal range: (0.93-1.7) d)   Na+ = 125 mEq/L e)   K+ = 2.4 mEq/L
  • 77. This ECG shows sinus bradycardia (52 bpm) with T wave flattening and very prominent U waves, best seen in leads V2-V3, most consistent with severe hypokalemia and/or certain types of drug effect/toxicity. The patient had marked hypokalemia. A low TSH with an elevated T4 level would be consistent with severe hyperthyroidism, which is associated with resting sinus tachycardia (or atrial fibrillation). Digoxin toxicity may be associated with sinus bradycardia but the QT interval would be relatively short. Hyponatremia and hyperchloremia are not associated with specific ECG findings. A long QT-U syndrome, hereditary or acquired, as present here, is the key ECG marker of risk for syncope or sudden cardiac death due to torsade(s) de pointes type ventricular tachycardia.
  • 78. 47 year old woman with a history of breast cancer and a life-threatening problem. Echocardiogram showed normal ventricular function and no pericardial effusion. This ECG is most consistent with which ONE of the following diagnoses?     a)   Hypercalcemia b)   Hyperkalemia c)   Digitalis toxicity d)   Tricyclic antidepressant overdose e)   Systemic hypothermia
  • 79. The clinical diagnosis was hypercalcemia (Ca++ 15.8 mg/dL) due to metastatic breast cancer. The ECG shows resting sinus tachycardia with borderline left axis deviation. There is also borderline low limb lead voltage. The key finding is the very short <300-320 msec) QT interval (QT=280 msec) such that the T wave appears to take off dirrectly from the J point with marked abbreviation of the ST segment. This finding correlates with shortening of ventricular action potential phase 2 duration. Elevation of the J point in leads V1-V2/V3 is sometimes also seen with hypercalcemia, simulating ischemia.The lower limits of the absolute QT and rate-corrected QT (QTc) intervals are not well defined, and relative shortening occurs physiologically at higher sinus rates. The two major pathologic causes of an acquired relatively short QT are hypercalcemia and digitalis glycoside effect. The latter is excluded here by the absence of characteristic &quot;scooped&quot; ST depressions. A hereditary form of short QT syndrome has been described recently in families with a history of sudden cardiac death.
  • 80.  
  • 81. 46 year-old man is found unconscious. What is the rhythm?    a)   Polymorphic (torsade-type) ventricular tachycardia b)   Atrial fibrillation with aberration c)   Sinus tachycardia with intermittent runs of monomorphic ventricular tachycardia d)   Multifocal atrial tachycardia with aberration e)   Sinus tachycardia with intermittent right bundle branch block
  • 82. This ECG shows a relatively regular, wide complex polymorphic ventricular tachycardia, rate 150 bpm, that is most likely a type of torsade(s) de pointes. See leads V4-V6 for classic “turning of the points” morphology. Intermittent ventricular fusion beats may be present. Causes of this life-threatening arrhythmia include a variety of drugs and specific electrolyte abnormalities (especially hypokalemia and hypomagnesemia) associated with long QT (U) intervals, as well as hereditary long QT syndromes due to “channelopathies.”Admission laboratory testing revealed a serum potassium of 2.2 (normal range 3.5-5.3 mEq/L), sodium of 143 (range 135-148 mEq/L), bicarbonate of 16 (normal 21-30mEq/L), phosphate 1.0 (range 2.7-4.5 mg/dL) and magnesium 2.5 (range 1.6-2.6 mg/dL.)
  • 83. 30 year-old man admitted with coma. This ECG is most consistent with which of the following abnormalities?     a)   Tricyclic antidepressant overdose b)   Myxedema c)   Systemic hypothermia d)   Brugada pattern e)   Hypokalemia
  • 84. This ECG shows classic signs of systemic hypothermia with sinus bradycardia (at about 52 beats/min), prominent J (Osborn) waves and QT prolongation (540 msec). The patient had a body temperature of 26.4 degrees C (79.5 degrees F). He also had hyperkalemia (serum K+ 6.9 mEq/L that may account for the prominent T waves superimposed on the hypothermia pattern and probably on an underlying physiologic early repolarization variant in this young adult man.
  • 85. What abnormality is suggested by the ECG in this 71 year old man?    a)   Hyperkalemia b)   Digitalis toxicity c)   Systemic hypothermia d)   Hypercalcemia e)   Tricyclic antidepressant overdose
  • 86. This ECG shows sinus rhythm at a rate of about 70 bpm (there is sinus arrhythmia with a short sinus pause at the beginning). Of particular note is the shortened Q-T interval (about 280 msec) and markedly abbreviated ST segment consistent with hypercalcemia. Subtle “doming” of the ST-T complex in leads V1-V3 with slight J point/ST elevations in the right chest leads is also sometimes seen in this setting. The patient’s serum calcium at the time of this ECG was very elevated at 14.8 mg/dL (normal range 8.4-10.2). After extensive evaluation, a diagnosis of primary hyperparathyroidism was made.
  • 87. 47 year old woman with a history of breast cancer and a life-threatening problem. Echocardiogram showed normal ventricular function and no pericardial effusion. This ECG is most consistent with which one of the following diagnoses?     a)   Acute subarachnoid bleed b)   Acute pulmonary embolism c)   Acute anterior myocardial infarction (MI) d)   Acute pancreatitis e)   Hypercalcemia
  • 88. The ECG shows a number of salient findings including: sinus tachycardia, long QT/QTc (.34 sec/.46), low limb lead and lateral QRS voltages, along with non-diagnostic slow R wave progression V1-V3 and non-specific ST-T changes. The only diagnosis listed above that accounts for the major findings is acute pancreatitis. The long QT (ST segment component is stretched out) is due to hypocalcemia (Ca++ 7.2 mg/dL with serum albumin of 2.7 g/dL). Sinus tachycardia is due to neuroautonomic activation in this context and the low voltage is consistent with ascites/anasarca.
  • 89. A 43 year-old man is found unresponsive. What is the most likely diagnosis?    a)   Hyponatremia b)   Brugada pattern c)   Tricyclic antidepressant overdose d)   Systemic hypothermia e)   Myxedema
  • 90. The rhythm is sinus bradycardia at a rate of about 46 bpm with prominent “J” (Osborn) waves in leads V4-V6 and marked QT prolongation (620 msec). The baseline noise in this context is consistent with shiver-related artifact. This combination of findings is virtually pathognomonic of systemic hypothermia. (The precordial voltage is borderline prominent without definite evidence of left ventricular hypertrophy.)This patient was a homeless man with no known medical history who was found unresponsive with a body temperature of 74.6 degrees F during an extremely cold spell. Laboratory findings revealed a low serum potassium level of 3.2 (normal range 3.5-5.0 mEq/L) that may have contributed to the repolarization prolongation, low serum calcium of 7.3 (normal range 8.4-10.2 mg/dL) proportional to the low serum albumin of 2.7 g/dL, and low normal serum magnesium 1.7 (range 1.6-2.6 mg/dL). A transthoracic echocardiogram showed a normal left ventricular ejection fraction of 60 % and symmetric, diastolic left ventricular wall thickness (septum and posterior wall) at the upper limits of normal (11 mm)
  • 92. Patient developed torsade de pointes subsequently. Note marked QT prolongation (QT around 580 msec) in patient on erythromycin and haloperidol.
  • 93. Why is this young cocaine abuser in the emergency department ?  
  • 94. He is having an evolving anteroseptal myocardial infarction secondary to cocaine. There are Q waves in V2-V3 with ST segment elevation in V2-V5 associated with T-wave inversion. Also noted are biphasic T-waves in the inferior leads. These multiple abnormalities suggest occlusion of a large left anterior descending (LAD) artery that wraps around the apex of the heart (or multivessel coronary artery disease). Left anterior oblique caudal angiographic projection demonstrating total occlusion of the mid-LAD (open arrow) distal to the takeoff of a large first diagonal branch. Also noted is a 90% tubular stenosis of the first obtuse marginal branch (closed arrow).
  • 95. A 77 year old man had a post-operative ECG following left lower lobectomy for lung cancer. Prior ECG was normal. He had no chest discomfort or other cardiac symptoms. He was on a continuous epidural infusion of bupivacaine. Echocardiogram showed normal ventricular wall motion and serial CK-MB enzyme tests were normal. The ECG is most consistent with which ONE of the following diagnoses?     a)   Acute pericarditis b)   Acute anterior ST elevation ST elevation MI c)   Acute right ventricular ST elevation MI d)   Brugada-type pattern, drug-induced e)   Benign early repolarization
  • 96. The ECG shows sinus rhythm with unusual ST segment elevations/T wave inversions (“coved” pattern) in V1 and V2. This morphology and localization is strongly suggestive of a Brugada-type pattern, which may be induced by certain drugs, especially sodium channel blocking agents, in susceptible individuals. The negative enzymes, normal echocardiogram and normalization of the ECG off the epicardial bupivacaine (a reported cause of this finding) are all consistent with the diagnosis drug-induced Brugada pattern. Failure to recognize this finding could result in inappropriate therapy for ST elevation MI or to development of life-threatening ventricular arrhythmias related to the Brugada pathophysiology.